Teoría de Números

87
TEORÍA DE NÚMEROS D. Francisco Medrano con series de ejercicios corregidos 1

description

Este curso corresponde a la asignatura con el mismo nombre dada durante el semestre de otoño 2013 en la Universidad de Ginebra.

Transcript of Teoría de Números

TEORÍA DE NÚMEROS

D. Francisco Medrano

con series de ejercicios corregidos

1

Índice

1. Divisibilidad 51.1. Notaciones . . . . . . . . . . . . . . . . . . . . . . . . . . . . 51.2. Algoritmo de la división . . . . . . . . . . . . . . . . . . . . 51.3. Máximo común divisor (mcd) . . . . . . . . . . . . . . . . . 51.4. El Algoritmo de Euclides . . . . . . . . . . . . . . . . . . . . 51.5. Teorema fundamental de la Aritmética . . . . . . . . . . . . 61.6. Serie de ejercicios 1 . . . . . . . . . . . . . . . . . . . . . . . 7

2. Funciones aritméticas 82.1. Funciones multiplicativas . . . . . . . . . . . . . . . . . . . 82.2. La función ϕ de Euler . . . . . . . . . . . . . . . . . . . . . . 92.3. La función de Möbius . . . . . . . . . . . . . . . . . . . . . . 102.4. Producto de convolución . . . . . . . . . . . . . . . . . . . . 112.5. Las funciones τ y σ . . . . . . . . . . . . . . . . . . . . . . . 132.6. Números perfectos . . . . . . . . . . . . . . . . . . . . . . . 142.7. Función zeta de Riemann . . . . . . . . . . . . . . . . . . . . 152.8. Serie de ejercicios 2 . . . . . . . . . . . . . . . . . . . . . . . 16

3. Congruencias 183.1. Teorema chino del resto . . . . . . . . . . . . . . . . . . . . . 183.2. Teoremas de Fermat y Euler . . . . . . . . . . . . . . . . . . 193.3. Teorema de Wilson . . . . . . . . . . . . . . . . . . . . . . . 203.4. Teorema de Lagrange . . . . . . . . . . . . . . . . . . . . . . 203.5. Raíces Primitivas . . . . . . . . . . . . . . . . . . . . . . . . 223.6. Indices (Logaritmos discretos) . . . . . . . . . . . . . . . . . 243.7. Esquema de Diffie-Helman . . . . . . . . . . . . . . . . . . . 253.8. Serie de ejercicios 3 . . . . . . . . . . . . . . . . . . . . . . . 25

4. Residuos cuadráticos 264.1. Símbolo de Legendre . . . . . . . . . . . . . . . . . . . . . . 264.2. Criterio de Euler . . . . . . . . . . . . . . . . . . . . . . . . . 274.3. Lema de Gauss . . . . . . . . . . . . . . . . . . . . . . . . . . 284.4. Ley de reciprocidad cuadrática . . . . . . . . . . . . . . . . . 294.5. Serie de ejercicios 4 . . . . . . . . . . . . . . . . . . . . . . . 314.6. Símbolo de Jacobi . . . . . . . . . . . . . . . . . . . . . . . . 314.7. Serie de ejercicios 5 . . . . . . . . . . . . . . . . . . . . . . . 33

2

5. Formas cuadráticas 355.1. Suma de cuadrados . . . . . . . . . . . . . . . . . . . . . . . 365.2. Serie de ejercicios 6 . . . . . . . . . . . . . . . . . . . . . . . 385.3. Sumas de cuatro cuadrados . . . . . . . . . . . . . . . . . . . 385.4. Serie de ejercicios 7 . . . . . . . . . . . . . . . . . . . . . . . 40

6. Aproximaciones diofánticas 426.1. Teorema de Dirichlet . . . . . . . . . . . . . . . . . . . . . . 426.2. Fracciones continuas . . . . . . . . . . . . . . . . . . . . . . 446.3. Serie de ejercicios 8 . . . . . . . . . . . . . . . . . . . . . . . 476.4. Teorema de Liouville . . . . . . . . . . . . . . . . . . . . . . 476.5. Números trascendentales . . . . . . . . . . . . . . . . . . . . 486.6. Serie de ejercicios 9 . . . . . . . . . . . . . . . . . . . . . . . 51

7. Cuerpos cuadráticos 567.1. Cuerpo de números . . . . . . . . . . . . . . . . . . . . . . . 567.2. Cuerpos cuadráticos . . . . . . . . . . . . . . . . . . . . . . . 587.3. Unidades . . . . . . . . . . . . . . . . . . . . . . . . . . . . . 597.4. Serie de ejercicios 10 . . . . . . . . . . . . . . . . . . . . . . 617.5. El cuerpo Gaussiano Q(i) . . . . . . . . . . . . . . . . . . . . 617.6. Ecuaciones de Pell . . . . . . . . . . . . . . . . . . . . . . . . 647.7. Serie de ejercicios 11 . . . . . . . . . . . . . . . . . . . . . . 66

8. La conjetura ABC 678.1. ABC para los polinomios . . . . . . . . . . . . . . . . . . . . 678.2. Preparativos para la prueba del teorema ABC para los poli-

nomios . . . . . . . . . . . . . . . . . . . . . . . . . . . . . . 698.3. ¿ABC para los enteros? . . . . . . . . . . . . . . . . . . . . . 70

9. Soluciones e indicaciones de las series de ejercicios 739.1. Serie de ejercicios 1 . . . . . . . . . . . . . . . . . . . . . . . 739.2. Serie de ejercicios 2 . . . . . . . . . . . . . . . . . . . . . . . 749.3. Serie de ejercicios 3 . . . . . . . . . . . . . . . . . . . . . . . 769.4. Serie de ejercicios 4 . . . . . . . . . . . . . . . . . . . . . . . 779.5. Serie de ejercicios 5 . . . . . . . . . . . . . . . . . . . . . . . 789.6. Serie de ejercicios 6 . . . . . . . . . . . . . . . . . . . . . . . 799.7. Serie de ejercicios 7 . . . . . . . . . . . . . . . . . . . . . . . 819.8. Serie de ejercicios 8 . . . . . . . . . . . . . . . . . . . . . . . 829.9. Serie de ejercicios 9 . . . . . . . . . . . . . . . . . . . . . . . 839.10. Serie de ejercicios 10 . . . . . . . . . . . . . . . . . . . . . . 859.11. Serie de ejercicios 11 . . . . . . . . . . . . . . . . . . . . . . 86

3

Contenido

4

1. Divisibilidad

1.1. Notaciones

Con el fin de fijar las ideas: N = {1,2,3, ...}, Z = {...,−1,0,1,2, ...} y decimosque a|b (léase “a divide b”) si por definición existe c ∈ N tal que b = ac.

Observación 1. Una consecuencia directa de la definición de divisibilidades que si c|a y a+ b = d, entonces a|b⇔ c|d.

1.2. Algoritmo de la división

Proposición 1. Para cada a,b ∈ Z, b > 0, ∃ q,r ∈ Z tales que a = bq + r con0 6 r < b.

Demostración. Podemos suponer que a > 0. Pongamos q := max{s ∈ Z : bs ≤a} y r := a− bq. Se verifica fácilmente, por definición de q y r, el enunciadode la proposición.

1.3. Máximo común divisor (mcd)

Definición 1. Decimos que d es un divisor común de a y b si d|a y d|b. Lla-mamos máximo común divisor de a y b, que notamos por mcd(a,b) o simple-mente por (a,b), al más grande de tales divisores comunes.

Proposición 2. Sea d = mcd(a,b), entonces d es el entero positivo más pequeñobajo a forma ax+ by con x,y ∈ Z.

Demostración. Sea D = ax + by > 0, vemos que d|D y entonces d ≤ D. Su-pongamos ahora que D es minimal (bajo esa forma), por la proposición 1escribamos a = Dq + r, entonces r = a−Dq = a(1− xq) + b(−yq). Pero r < D,como D es minimal entonces r = 0, lo que implica que D |a. De la mismamanera se muestra que D |b, de esta forma hemos mostrado que D es undivisor común de a y b, por definición D ≤ d y entonces D = d.

1.4. El Algoritmo de Euclides

Dados a y b enteros, el algoritmo siguiente nos permite encontrar el mcd(a,b)por divisiones euclidianas sucesivas.

Se calculan sucesivamente los cocientes y restos de las siguientes divisio-nes:

5

a = bq1 + r1 0 ≤ r1 < b,b = r1q2 + r2 0 ≤ r2 < r1,...

rk−2 = rk−1qk + rk 0 ≤ rk < rk−1,

rk−1 = rkqk+1 + rk+1 0 ≤ rk−1 < rk .

Observemos que r1 > r2 > ... ≥ 0 implica que este algoritmo se detiene enun número finito de etapas. Entonces, existe un k tal que rk+1 = 0.

Afirmación: mcd(a,b) = rk.

En efecto, viendo las igualdades precedentes de abajo hacia arriba se ob-serva fácilmente que rk |a y rk |b. De un modo similar, viendo las mismasigualdades esta vez de arriba hacia abajo, vemos que cada divisor comúnde a y b divide también rk, lo que permite concluir que mcd(a,b) = (a,b) =rk.

1.5. Teorema fundamental de la Aritmética

Definición 2. p ∈ N es llamado número primo o simplemente primo si p , 1y sus únicos divisores son 1 y p.

Lema 1. Si p es primo y p | ab, entonces p | a o p | b.

Demostración. Supongamos que p - a, entonces mcd(p,a) = 1 y luego exis-ten x,y ∈ Z tales que px+ay = 1. Multiplicando por b de ambos lados de laigualdad: bpx+ aby = b, luego como p | ab tenemos que p | b.

Teorema 1. (Teorema fundamental de la Aritmética). Podemos escribir n ∈N, n > 1 como producto de números primos n = pj11 · · ·p

jkk .

Demostración. Existencia: Sea n > 1, tomemos p1 el divisor de n distintode 1 más pequeño que n (que puede repetirse), p1 es entonces primo. Pon-gamos n′ = n

p1y luego repitamos. La descomposición en factores primos

queda entonces demostrada.

Unicidad: Supongamos dos descomposiciones de n = pj11 · · ·p

jkk = qi11 · · ·q

ill

en factores primos. Como pi | qi11 · · ·q

ill , por el lema 1, pi | qm para un cierto

m ∈ {1, · · · , l}, luego como pi y qm son primos se tiene que pi = qm. Lue-go dividimos n por pi = qm y repetimos este proceso, lo que demuestra launicidad.

6

Observación 2. Para encontrar el mcd de a y b, consideramos sus des-composiciones en factores primos (añadiendo los factores primos de a a by viceversa por medio de potencias de cero si es necesario) a = pn1

1 · · ·pnkk ,

b = pm11 · · ·p

mkk , 0 ≤ ni ,mi i = 1, · · · , k y luego la fórmula es:

mcd(a,b) =k∏i=1

pmın(ni ,mi )i

Teorema 2. Existe una infinidad de números primos.

Demostración. Supongamos que p1, · · · ,pn son los n primeros números pri-mos. Pongamos x = p1 · · ·pn + 1. Vemos que pk - x, k = 1, · · · ,n, de otra for-ma existiría pi con i ∈ {1,2, ...,n} tal que pi | x = p1...pn + 1 y por lo tantopi | (p1...pn + 1) − p1...pn = 1 lo que es una contradicción . Ahora bien, elteorema fundamental de la Aritmética nos dice que existe un primo q quedivide x, este primo es estrictamente más grande que cualquier pi de lalista {p1, · · · ,pn}. Luego siempre podremos encontrar un primo más grandeque uno dado, lo que muestra la infinitud de los números primos.

1.6. Serie de ejercicios 1

1. Mostrar que existe una infinidad de primos de la forma 4k + 3.

2. Un número primo de la forma 22n + 1 se llama número primo de Fer-mat. Mostrar que si 2n+1 es primo, entonces es un primo de Fermat.

3. (i) Mostrar que si m > n ≥ 0, entonces mcd(22m + 1,22n + 1) = 1. Indi-cación: Escribir 22m + 1 = 22m − 1 + 2.Sea pn el n-ésimo número primo y π(x) el número de primos com-prendidos entre 2 y un número x dado (que no es necesariamenteentero), es decir π(x) es una función que indica cómo se encuentranrepartidos los números primos.

(ii) Deducir de (i) que pn ≤ 22n + 1 y que π(x) ≥ loglogx para x ≥ 2.

4. Sean a,b,c ∈ N≥2. Mostrar que:

mcd(mcm(a,b),mcm(a,c),mcm(b,c)) = mcm(mcd(a,b),mcd(a,c),mcd(b,c))

5. Mostrar que el n-ésimo número armónico Hn :=∑nk=1

1k no es entero

para n ≥ 2

7

2. Funciones aritméticas

Definición 3. Una función aritmética es una función de N a valores en R obien en C.

Recordemos que llamamos parte entera de x ∈ R, que notamos por [x], alúnico entero que verifica x−1 < [x] ≤ x. La cantidad x− [x], se llama partefraccionaria de x y se escribe {x}.

Lema 2. Sea p primo, n ∈ N. Escribamos l = max{j ≥ 0 : pj | n!}. Entoncesl ≤

[np−1

].

Demostración. Observemos que entre los números 1, · · · ,n hay[np

]que son

divisibles por p (considerar la lista de los múltiplos de p: p,2p,...,kp con

kp ≤ n),[np2

]divisibles por p2, etc. No es difícil de demostrar que ∀ x,y ∈

R, [x+ y] ≥ [x] + [y]. Entonces

l =n∑

m=1

∞∑j=1, pj |m

1 =∞∑j=1

n∑m=1, pj |m

1 =∞∑j=1

[n

pj

]≤

∞∑j=1

n

pj

=[n

p − 1

].

La primera igualdad determina l contando sobre cada factor m de n! los jtales que pj dividem, luego en la segunda igualdad podemos intercambiarlas sumas ya que ambas son finitas (aunque una de ellas esté indexada has-ta el infinito, esta claro que es finita), la tercera igualdad resulta de contarlos números entre 1 y n que son divisibles por pj , luego en la desigual-dad se utiliza la propiedad de la función parte entera vista más arriba ymayoramos finalmente por una suma geométrica.

2.1. Funciones multiplicativas

Definición 4. Sea f : N→ R una función aritmética. Decimos que f es mul-tiplicativa si f (mn) = f (m)f (n), ∀m,n ∈ N tales que (m,n) = 1.

Observación 3. Si f . 0 y si f es multiplicativa, entonces f (1) = 1. Enefecto, f (1) = f (1 · 1) = f (1)2⇒ f (1) = 0 o f (1) = 1, la primera posibilidadse descarta ya que f . 0 (de hecho f (1) = 0 permite mostrar, por multipli-cabilidad de f , que f ≡ 0 lo que es contrario a la hipótesis).

Observación 4. Por multiplicabilidad, es suficiente conocer f sobre laspotencias de los números primos para conocer f en todas partes.

8

Lema 3. Sea f : N→ R multiplicativa. Entonces g(n) =∑d|nf (d) es también

multiplicativa.

Demostración. Sean m y n primos entre si (es decir (m,n) = 1), entonces

g(mn) =∑d|mn

f (d)(m,n)=1

=∑d|m

∑d′ |n,(d,d′)=1

f (dd′)(d,d′)=1

=∑d|m

f (d)∑d′ |n

f (d′)

= g(m)g(n).

2.2. La función ϕ de Euler

Definición 5. Para n ≥ 1, la función ϕ de Euler se define por ϕ(n) := {1 ≤j ≤ n : (j,n) = 1}.

Esta función es un ejemplo de funciones multiplicativas como se puedever fácilmente gracias al lema siguiente:

Lema 4.

ϕ(n) = n ·∏p|n

(1− 1

p

)Demostración. Si n = pα1

1 · · ·pαkk , entonces:

n ·∏p|n

(1− 1

p

)= n−

∑r

npr

+∑r>s

nprps

−∑r>s>t

nprpspt

+ · · · (1)

Ahora bien, se puede concluir la demostración con el principio de inclusión-exclusión o bien mediante el siguiente argumento:

Observemos que npr

es el número de enteros positivos más pequeños oiguales a n divisibles por pr , entonces

(1) =n∑

m=1

1−∑r,pr |n

1 +∑

r>s,prps |n1∓ · · ·

=

n∑m=1

(1−

(l(m)

1

)+(l(m)

2

)∓ · · ·

)︸ ︷︷ ︸

:=b(m)

9

donde l(m) = |{p primo : p | n}|. Si l(m) = 0 entonces b(m) = 1 y si l(m) > 0entonces b(m) = (1 − 1)l(m) = 0. Ahora bien (1) = |{m : l(m) = 0}| = ϕ(n) yaque l(m) = 0⇔ (m,n) = 1.

Lema 5. ∑d|nϕ(d) = n.

Demostración. Observemos que la función identidad id : N→ N es multi-plicativa y que el miembro de izquierda de la igualdad lo es también porel lema 3. Entonces, por la observación 4 sera suficiente demostrar estaigualdad para las potencias de primos pj :∑d|pj

ϕ(d) = 1 +ϕ(p) +ϕ(p2) + · · ·+ϕ(pj) lema 4= 1 + (p−1) + · · ·+ (pj −pj−1) = pj .

2.3. La función de Möbius

Definición 6. La función de Möbius µ está definida por:

µ(n) :=

1 si n = 10 si ∃ p primo tal que p2 | n

(−1)k si n = p1 · · ·pk

Se verifica directamente de la definición que µ es multiplicativa.

Lema 6. ∑d|nµ(n) =

{1 si n = 10 si no

Demostración. De nuevo, por el lema 3, como µ es multiplicativa, el miem-bro a la izquierda de la igualdad de arriba es una función multiplicativa(aplicada a n). De la misma forma el miembro a la derecha es una funciónmultiplicativa. Entonces será suficiente verificar esta igualdad sobre laspotencias de números primos pj .Para j = 0 es evidente. Si j > 0 tenemos:∑

d|pjµ(d) = µ(1) +µ(p) = 1− 1 = 0.

10

2.4. Producto de convolución

Sean f ,g : N→ C dos funciones aritméticas. El producto de convoluciónf ∗ g de f y g es la función aritmética definida por la fórmula:

(f ∗ g)(n) :=∑d|nf (d)g

(nd

).

Lema 7. Para todas f ,g,h funciones aritméticas se verifica:

(i) f ∗ g = g ∗ f(ii) (f ∗ g) ∗ h = f ∗ (g ∗ h)

(iii) f ∗ (g + h) = f ∗ g + f ∗ h

Demostración.

(i) (f ∗g)(n) =∑d|nf (d)g

(nd

)c:= n

d=∑dc=n

f (d)g(c), como esta última expresión es

simétrica en los divisores de n, se obtendrá similar expresión para (g∗f )(n).

(ii) f ∗(g ∗h)(n) =∑d|nf (d)

∑d′ | nd

h( ndd′

) c:= ndd′=

∑dd′c=n

f (d)g(d′)h(c), luego conclui-

mos como en (ii) por un argumento de simetría.

(iii) Trivial.

Proposición 3. Si f y g son multiplicativas, entonces f ∗ g es multiplicativa.

Demostración. Sean m y n tales que (m,n) = 1. Observemos que si d | m,d′ | n entonces (d,d′) = 1. Además todo c tal que c | mn es de la formac = dd′ con d |m y d′ | n. Luego

(f ∗ g)(mn) =∑d|mn

f (d)g(mnd

)=

∑d′ |n

∑d′′ |m

f (d′d′′)g( mnd′′d′

).

Donde, siguiendo la observación de arriba, hemos escrito d = d′d′′ cond′ | n y d′′ |m. Luego por multiplicabilidad de f y g se tiene

(f ∗ g)(mn) =∑d′ |n

∑d′′ |m

f (d′)f (d′′)g( md′′

)g( nd′′

)=

∑d′ |n

f (d)g( nd′

)∑d′′ |m

f (d′′)g( md′′

)= (f ∗ g)(n)(f ∗ g)(m).

11

Ejemplos

(1) La función aritmética definida por:

ε(n) :={

1 n = 10 sino

es una función multiplicativa (verificación trivial). Si f es una fun-ción aritmética cualquiera:

(f ∗ ε)(n) =∑d|nf (d)ε

(nd

)= f (n)⇒ f = f ∗ ε = ε ∗ f

es decir, ε es elemento neutro respecto a la operación ∗.

(2) La función aritmética 1, definida por 1(n) = 1 ∀ n es multiplicativa(de nuevo, la verificación es trivial).

(3) La función aritmética Id, definida por Id(n) = n ∀ n es multiplicativa.Tomemos ϕ la función de Euler, tenemos que:

Id(n) = n lema 5=∑d|nϕ(n) = (ϕ ∗1)(n)⇒ Id = ϕ ∗1

Sean a,b : N→ C dos funciones aritméticas (o si se prefiere, dos sucesionesen C).

Definición 7. Una serie de tipo∞∑n=0

a(n)ns

, se llama serie de Dirichlet.

De un curso de análisis complejo, no es difícil ver que el dominio de con-vergencia absoluta de una serie de Dirichlet es un semiplano abierto de C,limitado por una recta cuyos puntos tienen la misma abscisa. Este semi-plano puede ser vacío o todo el plano complejo C.

La multiplicación formal de dos series de Dirichlet ∞∑n=1

a(n)ns

∞∑n=1

b(n)ns

=∞∑n=1

c(n)ns

es otra serie de Dirichlet, donde c(n) =∑cd=n

a(c)b(d) = (a ∗ b)(n).

12

Proposición 4. ε = µ ∗1

Demostración. Como ε,µ y 1 son multiplicativas y por el lema 3, será sufi-ciente verificar esta igualdad para las potencias de primos pj , j ≥ 1:

(µ ∗1)(pj) =∑

0≤i≤jµ(pi)1(pj−i)︸ ︷︷ ︸

=1

= µ(1) +µ(p) + · · ·+µ(pj) = 0 = ε(pj).

Observación 5. Si f es una función aritmética cualquiera, entonces

g(n) :=∑d|nf (d) =

∑d|nf (d) · 1 =

∑d|nf (d)1

(nd

)= (f ∗1)(n).

Proposición 5. (fórmula de inversión de Möbius)Sean f : N→ C y g(n) =

∑d|n f (d), entonces

f (n) =∑d|nµ(d)g

(nd

).

Demostración. Por la observación 5 tenemos que g = f ∗ 1. Si convolucio-namos ambos miembros de esta igualdad por µ tenemos: g ∗µ = (f ∗1)∗µ =

f ∗ (1 ∗µ) = f ∗ (µ ∗1)prop 4

= f ∗ ε = f .

2.5. Las funciones τ y σ

τ(n) := |{d ∈ N : d | n}|

σ (n) :=∑d|nd

Por ejemplo τ(6) = 4 ya que los divisores de 6 son 1,2,3 y 6. Del mismomodo σ (6) = 1 + 2 + 3 + 6 = 12.

Propiedades

(1) τ(p) = 2⇔ p es primo.

(2) σ (p) = 1 + p⇔ p es primo.

(3) τ(n) = (1 ∗ 1)(n) =∑d|n1 y σ (n) = (Id ∗ 1)(n) =

∑d|nd. Es decir τ y σ

son funciones multiplicativas (proposición 3).

13

Por las propiedades precedentes, podemos calcular fácilmente el valor deσ :

n = pj11 · · ·pjkk ⇒ σ (n) = σ (pj11 ) · · ·σ (pjkk ) =

k∏i=1

pji+1i − 1pi − 1

, ya que σ (pj) = 1 + p +

p2 + · · ·+ pj =pj+1 − 1p − 1

si p es primo.

2.6. Números perfectos

Definición 8. Decimos que un número natural n es perfecto si σ (n) = 2n. Demanera equivalente, decimos que n es perfecto si es igual a la suma de todos susdivisores estrictamente menores que n.

Por ejemplo, 6 es perfecto ya que 6 = 1 + 2 + 3. También lo es 28, ya que28 = 1 + 2 + 4 + 7 + 14. El siguiente número perfecto es 496.

Ya en los tiempos de Euclides, se sabía que n = 2p−1(2p − 1) es perfectosi p y 2p − 1 son primos. Los números primos de la forma Nm = 2m − 1se llaman primos de Mersenne, de hecho se puede mostrar que si Nmes primo, entonces m es necesariamente primo. En efecto, si m no fue-se primo, entonces m = ab con a,b > 1, luego 2m − 1 = 2ab − 1 = (2a −1)

(1 + 2a + 22a + · · ·+ 2(b−1)a

)con cada uno de los factores entre paréntesis

estrictamente más grandes que 1, lo que contradice el hecho que Nm seaprimo.

Existen muchas preguntas sobre los números perfectos que aún no se hanresuelto, como por ejemplo: ¿Un número prefecto puede ser impar? ¿Existeun número infinito de números perfectos?

Teorema 3. (Euclides-Euler) Sea n par.

n es perfecto⇔ n = 2p−1(2p − 1) donde p y 2p − 1 son primos.

Demostración. ⇐ (Euclides):Sea n = 2p−1(2p−1), pongamos q = 2p−1. Tenemos que σ (n) = 1+2+22+· · ·+

2p−1+q+2q+ · · ·+2p−1q = (1+2+22 + · · ·+2p−1)(1+q) =2p − 12− 2

(1 + 2p − 1) =

2 · 2p−1(2p − 1) = 2n.

⇒ (Euler):Por hipótesis general, podemos escribir n = 2km, con k > 0 y m impar.

14

Tenemos que σ (2km) = σ (2k)σ (m) = (2k+1 − 1)σ (m). Por hipótesis n es per-fecto, es decir σ (n) = 2n, luego 2n = 2k+1m = (2k+1−1)σ (m)⇒ σ (m) = 2k+1l,donde l :=

m

2k+1 − 1. Distinguimos 2 casos:

l > 1En tal situación 1, l,m son algunos divisores distintos de m, por tantoσ (m) ≥ l +m+ 1 pero l +m = 2k+1l = σ (m) ¡contradicción!

l = 1σ (m) = 2k+1 = (2k+1 − 1)︸ ︷︷ ︸

=m

+1 = m + 1⇔ m es primo. Ahora bien m es

un primo de Mersenne, por lo tanto k + 1 es necesariamente primo,digamos p = k + 1. Finalmente n = 2km = 2p−1(2p − 1)

2.7. Función zeta de Riemann

La función zeta de Riemann, denotada por ζ, es la serie de Dirichlet aso-ciada a la sucesión constante a(n) = 1, es decir

ζ(s) :=∞∑n=1

1ns.

Ella converge para Re(s) > 1.

Teorema 4. (una fórmula de producto de Euler)

ζ(s) =∏

p primo

(1− p−s)−1 ,cuando Re(s) > 1.

Demostración. Sabemos que si |x| ≤ 1 entonces 1 + x + x2 + · · · = 11−x , luego∑∞

k=01psk

= 11−psk si Re(s) > 1.

Observemos que

N∑n=1

1ns≤

∏p≤N, primo

(1 +

1ps

+1p2s + · · ·+ 1

pNs

)

≤∏

p≤N, primo

(1

1− p−s

)≤

∏p primo

(1− p−s)−1 .

15

La parte derecha de esta desigualdad no depende de N . Ahora bien sitomamos N →∞, tenemos que la parte izquierda converge hacia ζ(s), esdecir

ζ(s) =∞∑n=1

1ns≤

∏p primo

(1− p−s)−1.

Para mostrar la otra desigualdad, observemos de nuevo que gracias al teo-rema fundamental de la aritmética se tiene

∏p primo, p≤N

(1 +

1ps

+ · · ·+ 1pMs

)≤∞∑n=1

1ns

= ζ(s).

En efecto, por la unicidad de la descomposición de un número en sus fac-tores primos, al desarrollar el producto de la parte izquierda de esta de-

sigualdad se obtienen fracciones del tipo1ms

con los m distintos.

Dejemos que los enteros N,M→∞. Se tiene entonces

∏p primo

(1− p−s)−1 ≤∞∑n=1

1ns

= ζ(s)

Corolario 1. Existe una infinidad de números primos.

Demostración. Supongamos que existe un número finito de números pri-mos. Si s = 1, ζ(1) es la serie armónica, la cual sabemos diverge. Pero comohay un números finito de primos, el producto de Euler

∏p(1 − p−1)−1 es

finito, lo que contradice el teorema 4.

2.8. Serie de ejercicios 2

1. Sea n ≥ 2 un entero con exactamente k factores primos distintos.Mostrar que

∑d|n|µ(d)| = 2k.

2. Dar una expresión de∑d|nµ(d)σ (d) en la que intervengan solamente

los factores primos de n.

16

3. Mostrar que∑n≤x

ϕ(n)n

=6π2x + O(logx), donde ϕ es la indicatriz de

Euler.Recordatorio: Decimos que f (x) =

x→aO(g(x)), si existen dos constantes

d > 0 y C tales que ∀x (|x − a| < d⇒ |f (x)| < C|g(x)|).

4. Sea Λ la función de Von Mangoldt, es decir la función aritméticadefinida por Λ(n) = logp si n = pk con k ≥ 1, Λ(n) = 0 sino. Calcular∑d|n

Λ(d) y expresar∑n≥1

Λ(n)ns

en función de ζ(s) y ζ′(s).

17

3. Congruencias

En la mayor parte de este capítulo se tratarán temas ya vistos probable-mente en cursos de Álgebra 1, por lo que nos limitamos a hacer un breverepaso de los principales resultados ya conocidos.

Definición 9. Sean x e y dos enteros, n un números natural. Decimos quex ≡ y mod n (léase x es congruente a y módulo n) si por definición existe k ∈ Ztal que x = y + kn.

Sabemos que si cocientamos el anillo Z con el ideal nZ, el resultadoes otro anillo Z/nZ cuyos elementos son {1, 2, · · · ,n− 1}, llamados clases derestos. El ideal nZ es maximal si y solamente si n es primo, lo que implicaque Z/nZ es un cuerpo si solamente si n es primo. Si p es primo, el cuerpoZ/pZ se escribe a menudo Fp. Para un anillo (A,+, ·) cualquiera, sabemostambién que los elementos invertibles respecto a la operación “·” formanun grupo, el que denotamos por A∗ o bien por U (A). En particular tenemosel grupo (Z/nZ)∗.

3.1. Teorema chino del resto

Nuestro objetivo es resolver ecuaciones lineales módulo n, n ∈ N. Porejemplo: encontrar un entero x tal que ax ≡ b mod n, es decir si existe xy por ende otro entero k tales que ax = b + kn o de manera equivalenteax+(−k)n = b. Ahora bien, por la proposición 2, sabemos que tales x y (−k)existen si y solamente si (a,n) | b. Supongamos que d = (a,n) divide b, enese caso la solución x existe y se puede calcular gracias al algoritmo deEuclides.

Un caso particular es cuando (a,n) = 1, en tal situación a es invertibleen Z/nZ y por lo tanto x = a−1b, luego levantamos un elemento x ∈ Z quepertenezca a la clase de x para obtener una solución de nuestro problemaoriginal. Observemos que x es una solución única módulo n.

Entonces para resolver la ecuación ax ≡ b mod n es necesario y suficien-te que d = (a,n) | b, en ese caso podemos dividir la ecuación ax + (−k)n = bpor d para obtener a′x+ (−k)n′ = b′, donde a′ = n

a , n′ = nd y b′ = n

b . Entonces(a′,n′) = 1 y el problema se reduce al caso particular.

18

Teorema 5. (Teorema chino del resto) Sean n1, · · · ,nk tales que (ni ,nj) = 1si i , j. Sean ci , · · · , ck ∈ Z. Entonces, existe una solución x del sistema:

x ≡ c1 mod n1x ≡ c2 mod n2

...x ≡ ck mod nk

Además, esta solución es única módulo n1 · · ·nk.

Demostración. Existencia:Sean n = n1 · · ·nk, Mi = n

nipara i = 1, ..., k. Por hipótesis (ni ,Mi) = 1 ⇒

∃ yi ∈ Z tal que yiMi = 1 mod ni . Pongamos x = y1M1c1 + · · · + ykMkck, noes difícil ver que tal x satisface x ≡ ci mod ni para i = 1, ..., k.

Unicidad:Supongamos que existe otro y tal que y ≡ ci ≡ x mod ni para i = 1, ..., k,entonces ni | y − x ∀ i ⇒ n1 · · ·nk = n | y − x ya que los nj son primos entresi dos a dos, luego n | y − x⇒ y ≡ x mod n.

Otra forma de presentar este resultado es, bajo las mismas hipótesis, laexistencia de un isomorfismo de anillos entre Z/ (n1Z∩ · · · ∩nkZ) y Z/n1Z×· · · ×Z/nkZ.

3.2. Teoremas de Fermat y Euler

Por definición, tenemos que

ϕ(n) = |{ elementos inversibles de Z/nZ}| = (Z/nZ)∗.

En efecto, ϕ(n) cuenta todos los elementos que son primos relativos con n,es decir todos los elementos inversibles de Z/nZ.

Proposición 6. (Euler 1760)

Si (a,n) = 1, entonces aϕ(n) ≡ 1 mod n.

Demostración. (a,n) = 1 ⇒ a ∈ (Z/nZ)∗ que es un grupo de orden ϕ(n),luego por el teorema de Lagrange de la teoría de grupos, el orden de a divideel orden del grupo, es decir aϕ(n) = 1⇒ aϕ(n) ≡ 1 mod n.

Esta prueba no es la dada por Euler ya que es “moderna” para su época,sin embargo es en esencia la misma. Un caso particular de esta proposiciónes el llamado:

19

Proposición 7. Pequeño teorema de Fermat (1640)

Sea p primo y a ∈ N, entonces ap ≡ amod p.

Demostración. si a ≡ 0 mod p, es trivial. Sino, a es invertible en Z/pZ, esdecir (a,p) = 1 luego por la proposición 6, aϕ(p) = ap−1 ≡ 1 mod p.

3.3. Teorema de Wilson

Proposición 8. n es primo⇔ (n− 1)! ≡ −1 mod p.

Demostración. ⇒)Supongamos que n > 2 (para n = 2 la proposición se verifica fácilmente) esprimo. Para 0 < a < p, a−1 existe módulo n.a = a−1 módulo n⇔ a2 = 1 módulo n⇔ a = 1,n−1. Los elementos restantes{2,3, ...,n− 2} pueden ser agrupados en parejas, cada pareja consiste en unnúmero y su inverso en Z/nZ. Entonces tenemos que:

2 · 3 · 4 · · ·n− 2 ≡ 1 mod n⇒ 1 · 2 · · · (n− 2)(n− 1) ≡ (n− 1) mod n

⇐)Supongamos por el absurdo que n = ab, con 1 < a,b < n− 1.

Si a , b, entonces ab | (n − 1)! ⇒ (n − 1)! ≡ 0 mod n lo que contradice lahipótesis.

Si n = a2 ⇒ 2a < n − 1, luego n = a2 | (n − 1)! y de nuevo se contradicela hipótesis.

3.4. Teorema de Lagrange

Consideremos un polinomio con coeficientes enteros f ∈ Z[X]. Estamosinteresados en la siguiente ecuación modular

f (x) ≡ 0 mod n.

Definición 10. El grado módulo n deg (f ) de un polinomio con coeficientesenteros es el m más grande tal que f (x) = amx

m + am−1xm−1 + · · · + a0, am .

0 mod n.

Contrariamente a lo que ocurre en un cuerpo, la ecuación f (x) ≡ 0 (n) pue-de admitir un número de soluciones superior al de su grado módulo n o

20

también no poseer ninguna. Por ejemplo, la ecuación 2x ≡ 4 (6) posee dossoluciones x = 2 y x = 5. Por el contrario, la ecuación 2x ≡ 5 (6) no poseeninguna ya que (2,6) = 2 - 5 (ver teorema chino del resto).

Ejemplos

(1) Observación: Si f (x) ≡ 0 mod n y que d | n, entonces f (x) ≡ 0 mod n.

Consideremos la ecuación f (x) = x2 + x + 7 ≡ 0 mod 130. Como enprincipio no tenemos un método general para resolver una tal ecua-ción, podríamos ensayar con todos los posibles valores de Z/130Z,lo cual no es una buena idea. Pero si podemos intentar mostrar queesta ecuación no posee ninguna solución. En efecto, como 2 | 130 yya que en Z/2Z sólo tenemos dos elementos {0,1}, es fácil ver quef (x) . 0 mod 2, entonces por la observación esta ecuación no poseesoluciones en el anillo Z/130Z.

(2) Resolver f (x) = x2 +x+ 7 ≡ mod 189 utilizando que f (x) ≡ 0 mod 27tiene como soluciones A = {4,13,12} y que f (x) ≡ 0 mod 7 tiene comosoluciones B = {0,6}.

Solución: 189 = 337, busquemos la solución del sistema{x ≡ a1 (27) a1 ∈ Ax ≡ a2 (7) a2 ∈ B

Como 27 y 7 son primos relativos, es decir (27,7) = 1, el teoremachino del resto se aplica. Para cada a1 ∈ A, a2 ∈ B tenemos la soluciónúnica módulo 189 dada por x = a1b17 + a2b227, donde b1 y b2 sonelegidos tales que b17 ≡ 1 (27) y b227 ≡ 1 (7), por ejemplo b2 = 4,b1 = −1 (si no se puede adivinar fácilmente estas soluciones, siemprees posible recurrir al algoritmo de Euclides). Las soluciones (en prin-cipio 6) son de la forma: x = a128 − a227, de hecho son exactamenteseis x = 13,49,76,112,139,175.

Este tipo de fenómenos no se producen cuando la ecuación poli-nomial es tratada en un cuerpo, ese es el enunciado principal delteorema de Lagrange.

Teorema 6. (Lagrange) Sea p primo, P ∈ Fp[X] (o bien con coeficientes en uncuerpo cualquiera), deg (P ) = d. La ecuación P (x) ≡ 0 mod p posee a lo muchod soluciones.

21

Demostración. Por inducción en d :

d = 0⇒ P = a0 . 0 (p), es decir sin soluciones.

d = 1 ⇒ P (X) = a1X + a0, con a1 . 0 (p). Ahora bien, esta ecuación notiene solución si (a1,p) - a0 y posee una solución si (a1,p) | a0, además estaes única (módulo p) ya que a1 es invertible en Fp.

Supongamos que el teorema se verifica para todo polinomio de grado mó-dulo d inferior o igual a k. Sea P ∈ Fp un polinomio de deg(P ) = k + 1.Supongamos además que P posee una raíz a (caso contrario no hay nadaque discutir), tenemos que:

P (X) = P (X)− P (a) =k+1∑i=0

ai(Xi − ai)

=k+1∑i=0

ai(X − a)(Xi−1 +Xi−2a+ · · ·+ ai−1)

= (X − a)G(X).

Luego, como deg(G) = k, por hipótesis de recurrencia G tiene a lo sumo kraíces. Al final, P tiene a lo sumo k + 1 raíces.

Así por ejemplo, Xp − 1 tiene p − 1 raíces en Fp. Por Fermat ap−1 ≡ 1 (p)∀ a ∈ {1,2,3, · · · ,p − 1}, es decir exactamente p − 1 raíces y todas diferentes.Este resultado nos permite dar otra demostración del teorema de Wilson:

Xp−1 − 1 = (X − 1)(X − 2) · · · (X − (p − 1)) = Xp−1 + · · ·+ (−1)p−1(p − 1)!

De nuevo consideremos p > 2, es decir un primo impar (el caso p = 2 severifica fácilmente), tenemos que (−1)p−1 = 1, luego comparando los coefi-cientes constantes: −1 ≡ (p − 1)! (p).

3.5. Raíces Primitivas

Definición 11. Sean a,n ∈ N tales que (a,n) = 1. El número d > 0 más pequeñoque satisface ad ≡ 1 (1) se llama orden de a (módulo n).

Observemos que el teorema 6 (Euler) implica que d | ϕ(n). En efecto (a,n) =1⇒ 1 ≡ aϕ(n) (n) ≡ aqd+r ≡ ar (n), 0 ≤ r < d ⇒ r = 0 por minimalidad de d,luego ϕ(n) = qd⇔ d | ϕ(n).

22

Pensando en términos de grupos, a ∈ (Z/nZ)∗ que es un grupo de ordenϕ(n). El subgrupo engendrado por a es de orden d, luego el teorema deLagrange de la teoría de grupos dice que el orden de ese subgrupo debedividir el orden del grupo.

Definición 12. a es primitivo si su orden (módulo n) es ϕ(n), es decir si a esde orden maximal. En términos de grupos, podemos re formular esta definicióncomo sigue: a es primitivo⇔ a engendra (Z/nZ)∗.

Lema 8. Si a es de orden d (módulo n), am es de orden d(d,m) (módulo n).

Demostración. Por división euclidiana es fácil ver que ak ≡ 1 (n)⇔ d | k.Sea j el orden de am, (am)j ≡ 1 (n)⇔ d |mj⇔ d

(d,m) |m

(d,m)j⇒d

(d,m) | j ya qued

(d,m) y m(d,m) son primos relativos. Entonces d

(d,m) ≤ j, pero por minimalidad

de j se tiene que d(d,m) = j.

Teorema 7. Sea p un primo impar. El número de elementos de orden d en Fpes ϕ(d), en particular existen primitivos ya que ϕ(p − 1) > 0.

Demostración. Por definición de orden (módulo p), podemos particionarlos elementos de F∗p = {1,2, · · · ,p−1} = Ad1

tAd2t· · ·tAdk , donde di son los

divisores de ϕ(p) = p − 1 y Adi = {a ∈ F∗p : a es de orden di}.

Sea ψ(d) = |Ad |,∑d|p−q

ψ(d) = p − 1. Supongamos que existe d (o bien di)

tal que ψ(d) , 0, sea entonces a ∈ Ad .

Afirmación: Los elementos de Ad son exactamente todos los am con 1 ≤m ≤ d, (m,d) = 1.

En efecto, todos los am con 1 ≤ m ≤ d son diferentes (por definición deorden de a), además (am)d ≡ (ad)m ≡ 1 (p), es decir los am son raíces de laecuación Xd − 1 = 0 en Fp, por el teorema 6 (Lagrange) esas son todas lasraíces. Así, todos los elementos de Ad son de la forma am y por el lema 8,am es de orden d si y solamente si (d,m) = 1.

La afirmación nos dice en particular que ψ(d) = ϕ(d), si suponemos Ad ,∅. Regresando al teorema, tenemos que

p − 1 =∑

d|p−1,ψ(d),0

ψ(d) =∑

d|p−1,ψ(d),0

ϕ(d).

23

Como ϕ∗1 = Id⇒∑d|p−1

ϕ(d) = p−1, luego no es posible que para un divisor

d de p−1 se tenga ψ(d) = 0. Entonces ψ(d) , 0∀ d | p−1⇒ ψ(d) = ϕ(d)∀ d |p − 1.

3.6. Indices (Logaritmos discretos)

Siguiendo la analogía con los números reales, decimos que x es el loga-ritmo discreto de c en base en a módulo n si por definición ax ≡ c mod n.Retomando notaciones familiares, escribiremos x = loga c o bien x = ind(c)sobreentendiendo la base a del logaritmo.

El cálculo de logaritmos discretos para n = p primo muy grande es un pro-blema difícil, sólo se conocen algoritmos subexponenciales como el IndexCalculus, es por eso que los logaritmos discretos juegan un papel impor-tante en la criptografía.

Ejemplos

(1) Trabajemos módulo p, p primo. El logaritmo discreto de 1 en basea ∈ F∗p está dado por x = p − 1. En efecto, es el teorema pequeño deFermat (proposición 7).

(2) Si x = 3 ⇒ 2x ≡ 3 (5), luego x = log2 3 = 3. Por otro lado, todas laspotencias de 4 son ya sea 1 o bien 4 módulo 5, entonces log4 3 noexiste.

Fijemos g primitivo módulo n. Para cada a ∈ (Z/nZ)∗ existe l tal que g l ≡a mod n, es decir l = logg a. Si otro l′ satisface g l

′ ≡ a mod n, entonces l yl′ son congruentes módulo ϕ(n). En adelante no escribiremos la base g dellogaritmo dándola por sobreentendida.

Proposición 9. Sean a,b ∈ (Z/nZ)∗, g un primitivo fijado.

(i) log1 = 0

(ii) logg = 1

(iii) loga+ logb = log(ab) mod ϕ(n)

Demostración. (iii) Como g es primitivo gϕ(n) ≡ 1 (n) ⇒ g log(ab) = ab =g logag logb = g loga+logb⇒ loga+ logb = log(a) mod ϕ(n).

24

3.7. Esquema de Diffie-Helman

Supongamos que Marco y Dani quieren intercambiar una clave parapoderla utilizar en un protocolo criptográfico cualquiera. Ellos se ponen deacuerdo en un número primo p y un elemento g ∈ F∗p que hacen público.Marco elige su clave secreta x ∈ F∗p, del mismo modo Dani elige la suyay ∈ F∗p. Marco calcula con su clave secreta X ≡ gx mod p, igualmente Danicalcula Y ≡ gy mod p. Marco le envía X a Dani y Dani le envía Y a Marcoa fin de poder intercambiarse una clave que cada uno calcula con su cla-ve secreta. Marco descifra la clave calculando K ≡ Y x mod p e igualmenteDani descifra la clave calculando K ′ ≡ Xy mod p.

Marco y Dani tiene la misma clave ya que

K ≡ Y x ≡ (gy)x ≡ (gx)y ≡ Xy ≡ K ′ modp

El espía Roberto observa lo que sucede (Roberto conoce p,g,X e Y ), élno puede obtener la clave intercambiada entre Marco y Dani ya que no escapaz de calcular logaritmos discretos en un tiempo razonable (puesto quepara obtener K o K ′ Roberto necesita conocer las claves secretas de Marcoo de Dani).

3.8. Serie de ejercicios 3

1. Mostrar que∞∑n=1

ϕ(n)xn

1− xn=

x

(1− x)2 .

2. Encontrar todos los enteros x tales que 2x ≡ 1(mod 3), 3x ≡ 1(mod 5)y 5x ≡ 1(mod 7).

3. Mostrar que para todos p primo, a, k ≥ 1 enteros se tiene akp−k+1 ≡a(mod p). Deducir que 798 | a19 − a.

4. Encontrar enteros a,b,c,d y e tales que para todo x entero, al menosuna de las congruencias x ≡ a (mod 2), x ≡ b (mod 3), x ≡ c (mod 4),x ≡ d (mod 6) y x ≡ e (mod 12) sea satisfecha.

25

4. Residuos cuadráticos

4.1. Símbolo de Legendre

Hace alrededor de 4000 años, los matemáticos como Al-Khwarizmi,sabían cómo resolver las ecuaciones de tipo ax2 + bx + c = 0 por medio detécnicas que ellos llamaban “Al-jabr, Al-muqabala” que podemos traducirpor “balanceo y complementación”. Cuando se trabaja en un cuerpo dife-rente de R, como por ejemplo Z/pZ, esta ecuación presenta la dificultadsiguiente: ¿es ∆ = b2 − 4ac un cuadrado módulo p? Suponiendo que p nodivide 4a.

Nos interesamos entonces en la ecuación modular x2 ≡ k mod p. Obser-vemos que:

Si p = 2⇒ x = 0 o x = 1 (la ecuación siempre tiene solución).

Si k = 0 y p es un primo cualquiera, x = 0 es la única solución.

Si x es una solución, entonces −x lo es también.

Para p , 2 se tiene que −x . x (p) y entonces, si conocemos una solu-ción x, por el teorema 6 (Lagrange) conocemos todas las solucionesya que la ecuación posee a lo sumo dos raíces.

Definición 13. Sean a y n tales que (a,n) = 1. Decimos que a es un residuocuadrático módulo n si la ecuación x2 ≡ a (n) posee una solución.

Definición 14. Sea p primo, el símbolo de Legendre se define por:

(ap

)=

0 si p | a1 si a ≡ x2(p)−1 si no

Ejemplos

(1)(a2

)= 1 ya que 02 = 0 y 12 = 1.

(2)(

1p

)= 1 ya que (±1)2 = 1.

26

4.2. Criterio de Euler

Proposición 10. Sea p > 2 primo. Hay p−12 residuos cuadráticos en F∗p. Ade-

más, estos residuos cuadráticos son 12,22, ...,(p−1

2

)2.

Demostración. Pongamos r = p−12 . Claramente 12,22, ...,

(p−1

2

)2son residuos

cuadráticos, además son todos diferentes. En efecto, supongamos que a2 ≡b2 (p)⇒ (a+ b)(a− b) ≡ 0 (p), luego como Z/pZ es un cuerpo, en particularun dominio integral, se tiene a+b ≡ 0 o bien a−b ≡ 0. Si a+b ≡ 0, tenemosque a+ b = p⇒ a = p − b > r para b ∈ {1, ..., p−1

2 }, luego a = b.

Mostremos ahora que {1, ..., p−12 } son todos los residuos cuadráticos. Cada

vez que y ≡ x2 (p), por el teorema de Fermat (teorema 7): yr ≡ x2r ≡ xp−1 ≡1 (p), luego los residuos cuadráticos satisfacen la ecuación yr − 1 ≡ 0 (p) ypor el teorema de Lagrange (teorema 6) existe a lo sumo p−1

2 soluciones adicha ecuación, las cuales conocemos todas.

Observación 6. Si g es un elemento primitivo, entonces 1, g2, g4, ..., g2(r−1)

es la lista de residuos cuadráticos.

Proposición 11. (Euler) Si p > 2 primo, entonces:(ap

)≡ a

(p−1

2

)mod p.

Demostración. Si a = 0 la igualdad es verificada. Pongamos como antesr = p−1

2 y distingamos dos casos:

(1) Si(ap

)= 1 existe x tal que a ≡ x2 (p), luego ar ≡ x2r ≡ xp−1 Fermat≡ 1 (p).

(2) Si(ap

)= −1 se tiene que ar . 1 (p). Por Fermat 1 ≡ ap−1 ≡ (ar)2⇒ ar ≡

±1 (p), luego ar ≡ −1 (p).

Corolario 2. (i)(abp

)=

(ap

)(bp

)

(ii) a ≡ b (p)⇒(ap

)=

(bp

)

27

(iii)(−1p

)= (−1)

p−12

Observación 7. No hay necesidad de considerar estas relaciones módulop.

4.3. Lema de Gauss

Definición 15. El representante más pequeño de amodn es el a′ tal quea′ ≡ amodn con −1

2n < a′ ≤ 1

2n.

Ejemplo

−2 es el representante más pequeño de 15 módulo 17.

Lema 9. (Gauss) Sea p > 2 primo y a tal que (a,p) = 1. Sean aj los represen-tantes más pequeños de j · a, para j = 1,2, ..., p−1

2 . Entonces:(ap

)= (−1)l ,

donde l = |{1 ≤ j ≤ p−12 | aj < 0}|.

Ejemplo p = 7, a = 2. Los representantes más pequeños modulo 7 de j · apara j = 1,2,3 son 2,−3 y −1, luego por el lema 9:

(27

)= (−1)2 = 1. Por

ejemplo 42 ≡ 2 (7).

Demostración. Afirmación: {|aj |}rj=1 = {1,2, ..., r}.

Prueba de la afirmación: Por definición 1 ≤ |aj | ≤ r. Mostremos que son to-dos diferentes:Caso 1: aj = −ak⇔ ak+aj = 0⇔ a(j+k) = 0

a,0⇒ k+j = 0 lo que no es posible

ya que 0 < k + j ≤ p.

Caso 2: aj = ak⇔ a(j − k) = 0a,0⇒ j = k.

De regreso al lema, gracias a la afirmación |a1| · · · |ar | = r! ⇒ a1 · · ·ar =r!(−1)l . Sabemos también que aj ≡ ja (p), luego a1 · · ·ar ≡ r!ar (p). Com-parando estas dos igualdades módulo p tenemos ar ≡ (−1)l (p), finalmente

por el criterio de Euler (proposición 11) tenemos(ap

)≡ ar ≡ (−1)l (p).

Corolario 3. (2p

)= (−1)

18 (p2−1).

28

4.4. Ley de reciprocidad cuadrática

Teorema 8. (Gauss 1976) Sean p,q dos primos diferentes e impares, entonces:(p

q

)(q

p

)= (−1)

p−12 ·

q−12 .

Este teorema, a veces llamado "teorema áureo" permite relacionar la solubi-lidad de dos congruencias de segundo grado: x2 ≡ p mod q, y2 ≡ q mod p.En efecto la fórmula de reciprocidad cuadrática nos permite ver cuándop y q son o no simultáneamente cuadrados módulo q y p respectivamen-te. Se conoce alrededor de 200 demostraciones diferentes de esta fórmula,nosotros examinaremos una de ellas que es bastante simple y original.

Demostración. La idea principal de esta demostración es la de utilizar ellema 9. Con una notación ligeramente diferente, pongamos j = x y aj =px − qy donde y se determina de las relaciones −1

2q < aj , 0 < x < 12q, es

decir y < pxq + 1

2 <12(p+ 1). Como y es entero, el lema 9 puede reformularse

como sigue: (p

q

)= (−1)l

l = |{(x,y) ∈ Z2 | 0 < x < 12q,0 < y <

12p tales que − 1

2q < px − qy < 0}|.

De manera completamente simétrica:(q

p

)= (−1)m

m = |{(x,y) ∈ Z2 | 0 < x < 12q,0 < y <

12p tales que − 1

2p < qy − px < 0}|.

Sobre el rectángulo D =[0, 1

2q]×[0, 1

2p]

de la figura 1. Las rectas px − qy =

0,px−qy+12q = 0 y −px+qy+1

2p no intersecan ningún punto en el interior dela red Z2⋂D. Los puntos enteros (x′, y′) dentro del triángulo superior delrectángulo D está en biyección con los puntos enteros (x,y) del triánguloinferior del rectángulo D vía las siguientes fórmulas:

x =12

(q+ 1)− x′, y =12

(p+ 1)− y′

Por ejemplo px′−qy′ = p(

12(q+ 1)− x

)−q

(12(p+ 1)− y

)= p

2−q2−px+qy ≤ −q2 .

29

Figura 1: Reciprodidad cuadrática

Finalmente, hemos mostrado quep − 1

2·q − 1

2− (l +m) es par, luego(

p

q

)(q

p

)= (−1)l+m = (−1)

p−12 ·

q−12

Ejemplo. Utilizando los corolarios 2 y 3 y el la ley de reciprocidad cuadrá-tica, mostrar que -42 es un cuadrado módulo 61.Solución: (−42

61

)=

(−161

)︸︷︷︸

1

( 261

)︸︷︷︸−1

( 361

)( 761

)

ya que(−1

61

)= (−1)

61−12 = 1 y

( 261

)= (−1)

18 (62)(60) = −1. Continuando de la

misma manera( 761

)=

(617

)(−1)

61−12 ·

7−12 =

(57

)=

(75

)(−1)

7−12 ·

5−12 =

(25

)= (−1)

18 (52−1) = −1.

y( 361

)=

(613

)(−1)30·1 =

(13

)= 1.

Finalmente(−42

61

)= 1 · (−1) · (−1) · 1 = 1.

30

4.5. Serie de ejercicios 4

1. Encontrar todos los enteros x tales que 3x ≡ 1(mod 5), 5x ≡ 1(mod 17)y 7x ≡ 1 (mod 23).

2. Sean a1, ..., ap y b1, ...,bp dos sistemas completos de residuos módulop, con p primo (un conjunto X = {x1, ...,xn} de n enteros es un sistemacompleto de residuos módulo n si para todo entero x ∈ Z existe un únicoxk ∈ X tal que xk ≡ x (mod n)). Mostrar que a1b1, ..., apbp no puede sernunca un sistema completo de residuos módulo p si p , 2.

3. Mostrar que, para p ≥ 3 primo, la congruencia x2 ≡ (−1)p+1

2 (mod p)admite por solución x =

(p−1

2

)!.

4. Sean a,n ≥ 2 dos enteros tales que an−1 ≡ 1(mod n) pero am . 1(mod n)para todo divisor m , n− 1 de n− 1. Mostrar que n es primo.

4.6. Símbolo de Jacobi

Una generalización del símbolo de Legendre es el símbolo de Jacobi.Sean n = p1 · · ·pk un número impar con los pi primos (no necesariamentediferentes) y a un entero, el símbolo de Jacobi se define por la siguientefórmula: (a

n

):=

∏ki=1

(api

)si (a,n) = 1

1 si n = 10 si (a,n) > 1

donde(api

)es el símbolo de Legendre.

Si n = p primo, el símbolo de Jacobi no es otra cosa que el símbolo deLegendre, una razón por la cual no utilizamos una nueva notación.

Observación 8. Si a es un cuadrado módulo n, es decir existe x tal que

x2 ≡ a mod n, entonces(an

)= 1. Pero contrariamente a lo que nos dice por

definición el símbolo de Legendre, si n no es primo, la afirmación recípro-ca para el símbolo de Jacobi, es en general falsa.

Ejemplo.( 215

)=

(23

)(25

)= (−1)(−1) = 1, sin embargo un cálculo a la mano

de los restos cuadráticos módulo 15 nos muestra que x2 ≡ 2 mod 15 notiene solución.

31

Proposición 12. (i)( amn

)=

( am

)(an

)(ii)

(abn

)=

(an

)(bn

)(iii)

(−1n

)= (−1)

n−12

(iv)(2n

)= (−1)

n2−18

(v) Si (m,n) = 1, entonces(mn

)( nm

)= (−1)

m−12 ·

n−12

Demostración. Observemos que el símbolo de Jacobi generaliza las pro-piedades ya conocidas para el símbolo de Legendre, las tres primeras sedemuestran directamente. Para mostrar (v) procedemos como sigue:

Observemos que si a y b son impares, entonces

ab − 12−(a− 1

2+b − 1

2

)=

(a− 1)(b − 1)2

≡ 0 mod 2. (2)

Supongamos que m =∏ri=1pi y n =

∏sj=1 qj , tenemos que

(mn

)=

r∏i=1

s∏j=1

(piqj

)por las propiedades (i) e (ii)

=r∏i=1

s∏j=1

(qjpi

)(−1)

pi−12 ·

qj−12 por reciprocidad cuadrática

=( nm

) r∏i=1

s∏j=1

(−1)pi−1

2 ·qj−1

2

=( nm

)(−1)

∑sj=1

∑ri=1

pi−12 ·

qj−12 .

Ahora solo queda mostrar que

r∑i=1

s∑j=1

pi − 12·qj − 1

2≡ m− 1

2· n− 1

2mod 2.

32

En efecto, aplicando la fórmula (2) las veces que sean necesarias:

r∑i=1

pi − 12≡p1p2 − 1

2+

r∑i=3

pi − 12

mod 2

≡p1p2p3 − 1

2+

r∑i=4

pi − 12

mod 2

...

≡∏ri=1pi − 1

2mod 2,

luegor∑i=1

s∑j=1

pi − 12·qj − 1

2=

r∑i=1

pi − 12

s∑j=1

qj − 1

2

∏ri=1pi − 1

2·∏sj=1 qj − 1

2≡ m− 1

2· n− 1

2mod 2.

Ya hemos visto que el si el símbolo de Jacobi de(an

)es 1, a no es necesaria-

mente un cuadrado módulo n. Sin embargo si(an

)= −1, entonces a no es

un cuadrado módulo n ya que a no lo es para algún primo pk que divide an.

4.7. Serie de ejercicios 5

1. Determinar todas las soluciones de la congruencia y2 ≡ 5x3 (mod 7).

2. Sean p un primo y k un entero positivo. Mostrar quep∑n=1

nk es con-

gruente a −1 módulo p si p − 1 divide k y a 0 sino.Indicación: Escribir la suma utilizando un generador del grupo F∗p.

3. Mostrar que(

2p

)= (−1)

p2−18 .

4. ¿Es 15 un cuadrado módulo 73?

33

5. Sea p = 4k+3 un primo tal que p′ = 2p+1 es también primo. Mostrarque 2p ≡ 1 (mod p′). Deducir que 2251 − 1 no es primo.Nota: Mn = 2n − 1 se llama número de Mersenne. Si Mn es primo, en-tonces se dice que Mn es un primo de Mersenne.

34

5. Formas cuadráticas

Definición 16. Una forma cuadrática q sobre R es una función

q(x1, ...,xn) =n∑i=1

n∑j=1

aijxiyj

con los aij ∈ Z.

Si ponemos x = (x1, ...,xn)t y A = (aij) ∈ Mat (Rn), podemos escribir de ma-nera más compacta q(x) = xtAx.

Decimos que q es definida positiva (respectivamente definida negativa)si para todo x , 0, q(x) > 0 (respectivamente si para todo x , 0, q(x) < 0).De manera similar se utilizan los términos semi-definida positiva o negati-vamente, simplemente cambiando la desigualdad estricta por un ≥ o un ≤respectivamente. En caso de no cumplirse ninguna de esas condiciones, sedice que la forma es indefinida.

El grupo de automorfimos de Zn, Aut(Zn) es

SLnZ = {M ∈Mat(Zn) |detM = 1}.

Dos formas q y q son equivalentes si por definición existe λ ∈ SLnZ tal queq = q ◦λ. En particular Im(q) = Im(q).

Sea A, A las matrices de las formas q y q respectivamente, entonces

A = λtAλ,

de donde se tiene de forma directa que detA = detA.

El caso n = 2 nos da la forma cuadrática f (x,y) = ax2 + bxy + cy2 cona,b,c ∈ Z. El discriminante d de f es por definición d := b2 − 4ac. Si a , 0tenemos que 4af (x,y) = (2ax+ by)2 −dy2. Si a,d , 0 los signos de a y d nosayudan a determinar si la forma es definida positiva (negativa) o no, porejemplo si a > 0 y d < 0 la forma es definida positiva. Si a < 0 y d < 0 laforma es definida negativa. Si d = 0 et signo de a determina si la forma eso no semi-definida positiva (negativa). Si d > 0 la forma es indefinida.

Asociamos a f la matriz

A =(a b/2b/2 c

).

35

Vemos que det A = −d/4, luego el signo del determinante de A está rela-cionado con la positividad de f .

5.1. Suma de cuadrados

La teoría aditiva de números es una rama de la teoría de números que estu-dia formas de representar números enteros positivos como sumas.

Si S ⊂ N, podemos preguntarnos qué subconjunto de N podemos repre-sentar si sumamos los elementos de S entre si. Por ejemplo, si nos permiti-mos sumar únicamente dos elementos de S a la vez, la pregunta es ¿a quécorresponde S + S?

Ejemplos famosos

(1) Si S = {02,12,22,32, ...}, Bachet conjeturó en 1621 que S+S+S+S = N,es decir que todo número natural se escribe como la suma de cuatrocuadrados. Lagrange demuestra esta conjetura en 1770.

(2) Si S = {2,3,5,7, ...} (los primos), Goldbach conjeturó en 1742 que todonúmero par ≥ 4 se escribe como la suma de dos primos. En nuestranotación S + S = { todos los pares ≥ 4}. Esta conjetura sigue sin de-mostrar, tampoco se la ha podido refutar ya que se ha podido com-probar por ordenador para pares muy grandes.

(3) Sigamos con S = {todos los primos}. Vinogradov conjetura que S +S + S = {todos los impares ≥ 7} y da una prueba incompleta demos-trando que es el caso para impares “muy grandes”, sin ser precisoen lo que se entiende por muy grande. En el 2013, Harald Helfgott,un matemático peruano, publica una demostración que actualmenteestá en curso de validación.

(4) Problema de Waring (1770): dados k ∈ N, Sk = {0k ,1k ,2k ,3k , ...} ¿existeun m tal que todo número natural se escribe como una suma de mtérminos de Sk? En nuestra notación ¿Sk + · · ·+ Sk︸ ︷︷ ︸

suma finita

= N?

Teorema 9. (Fermat-Euler) Sea n = 2αpk11 · · ·p

kss q

l11 · · ·q

lrr con pi , qj primos

diferentes tales que pi ≡ 3 mod 4 ∀i y qj ≡ 1 mod 4 ∀j. Entonces, n es suma dedos cuadrados si y sólo si todos los ki son pares.

36

Demostración. ⇒)

Si n = x2+y2 con x,y ∈ Z y sea p primo tal que p | n con p ≡ 3 mod 4. Enton-ces 0 ≡ x2 +y2 mod p⇔ x2 ≡ −y2 mod p. Podemos suponer que ya sea x o yno son divisibles por p, si no no hay nada que mostrar. Sin pérdida de gene-

ralidad supongamos que p - y, luego(xy

)2

≡ −1 mod p. Pero si p ≡ 3 mod 4,

-1 no es un cuadrado módulo p ya que(−1p

)= (−1)

p−12 = −1, luego nece-

sariamente x ≡ y ≡ 0 mod p⇒ n

p2 =(xp

)2

+(y

p

)2

⇒ p2 | n. Repetimos este

argumento para todos los primos p tales que p | n con p ≡ 3 mod 4.

⇐)

Observemos que (a2 + b2)(c2 +d2) = (ac− bd)2 + (ad − bc)2 (poner por ejem-plo z1 = a + ib, z2 = c + id y utilizar la identidad |z1|2|z2|2 = |z1z2|2). Estaidentidad es en particular cierta para todo a,b,c y d enteros, ella nos di-ce que sim y n son dos sumas de cuadrados, su productomn lo es también.

Ahora bien, 2 = 12 + 12 y los pi tales que pi | n con pi ≡ 3 mod 4 apare-cen, por hipótesis, en potencias pares ki = 2si y luego pkii = (psii )2 + 02. Así,sólo queda por mostrar que los primos q tales que q ≡ 1 mod 4 se escribencomo la suma de dos cuadrados q = x2 + y2.(−1q

)= (−1)

q−12 = 1 ya que q ≡ 1 (4), existe entonces x tal que −1 ≡ x2 (q).

Consideremos la función f (u,v) = u+xv y sea k =[√q]. Observemos quek <

√q < k+ 1 ya que q es primo. Consideremos las parejas (u,v) con 0 ≤ u,v ≤

k, tenemos en total (k + 1)2 de tales, luego (k + 1)2 > (√q)2 = q. Si consi-

deramos la función f a valores en Fq, es decir a la pareja (u,v) le asocia-mos el valor u + xv mod q, el principio de Dirichlet nos dice que existen(u′,v′) y (u,v) con 0 ≤ u′,v′,u,v ≤ k y (u′,v′) , (u,v) tales que f (u,v) ≡f (u′,v′) mod q, es decir u+xv ≡ u′+xv′ mod q⇒ u−u′ ≡ x(v′−v) mod q⇒(u −u′)2 ≡ x2(v′ − v)2 mod q⇒ (u −u′)2 + (v′ − v)2 ≡ mod q.

Pongamos a = u − u′, b = v′ − v, tenemos entonces a2 + b2 ≡ 0 mod q, esdecir q | a2 + b2, pero como (u,v) , (u′,v′) y a = u − u′ ≤ k < √q (de igualforma para b) tenemos 0 < a2 + b2 < 2q, por consecuencia q = a2 + b2.

37

5.2. Serie de ejercicios 6

1. Determinar los primos p para los cuales 5 es un cuadrado módulo p.

2. Calcular el símbolo de Jacobi(

123917

).

3. Sea p = 2n + 1 un primo. Mostrar que g es una raíz primitiva módulop si y solamente si, g no es un cuadrado módulo p. Deducir que sin > 1, 3 es una raíz primitiva módulo p.

4. Sea f (x) = ax2 + bx + c con a,b y c enteros y sea p un primo imparque no divide a a. Mostrar que el el número de soluciones de la con-gruencia f (x) ≡ 0 (mod p) es 1 +

(dp

), donde d = a2 − 4ac.

5. Mostrar que la aserción “todo número entero superior o igual a 6puede escribirse como la suma de tres números primos” es equiva-lente à la conjetura de Goldbach.

5.3. Sumas de cuatro cuadrados

Recordemos que Bachet enunció en 1621 su famosa conjetura: “todonúmero natural se puede escribir como la suma de cuatro cuadrados”, lacual fue demostrada más tarde en 1770 por Lagrange. Lagrange había in-cluso mostrado que no era posible hacer algo mejor, que por ejemplo 3cuadrados no eran suficientes: 7 = 22 +12 +12 +12. De manera más generalsi n ≡ 7 (8), entonces n no puede escribirse como la suma de tres cuadra-dos ya que, en particular, se tendría n ≡ x2

1 +x22 +x2

3 (8), pero los cuadradosmod 8 son x2 ≡ 0,1,4 (8), luego combinando estos restos cuadráticos en

tres sumandos obtenemos 0,1,2,3,4,5,6 pero no 7.

Para demostrar la conjetura de Bachet, necesitaremos una identidad si-milar a la que utilizamos para probar el teorema 9, dicha identidad laobtenemos de los cuaternionesH = {x+ iy+jz+kw |x,y,z,w ∈ R} con i, j y kverificando las relaciones i2 = j2 = k2 = −1 y ijk = −1, a partir de las cualespodemos deducir que ij = k y ji = −k, de donde vemos que con la estructu-ra de grupo multiplicativo H no es conmutativo. Como para los númeroscomplejos, es posible definir una especie de valor absoluto sobre H po-niendo |x+ iy+ jz+kw| :=

√x2 + y2 + z2 +w2. Un cálculo elemental permite

ver que este valor absoluto es multiplicativo, es decir si a,b ∈ H entonces|a||b| = |ab|, de donde |a|2|b|2 = |ab|2. Esta última relación sera nuestro puntode partida para demostrar el enunciado de Bachet.

38

De forma más explícita pongamos a = x+iy+jz+kw y b = x′+iy′+jz′+kw′,entonces

(x2 + y2 + z2 +w2)(x′2 + y′2 + z′2 +w′2) = (xx′ + yy′ + zz′ +ww′)2+

(xy′ −yx′ +wz′ −zw′)2 +(xz′ −zx′ +yw′ −wy′)2 +(x′w′ −wx′ +zy′ −yz′)2 (3)

Teorema 10. (Lagrange 1770) Todo n > 0 tiene la forma n = x2 +y2 +z2 +w2,con x,y,z,w ∈ N∪ {0}.

Demostración. La identidad (5.3) nos dice que es suficiente demostrar esteorema para n = p primo. Podemos incluso considerar p > 2 ya que 2 =12 + 12 + 02 + 02.

Sea entonces p > 2, para 0 ≤ x ≤ 12(p − 1) hemos visto por la proposición

10 que los x2 son todos distintos. Un argumento de traslación nos permiteconcluir que de modo similar los −1 − y2 para 0 ≤ y ≤ 1

2(p − 1) son todos

distintos, luego existen x,y tales que x2 = −1−y2⇒ x2+y2+1 < 1+2(

12p

)2<

1 + 12p

2 < p2.

x2 + y2 + 1 ≡ 0 mod p ⇒ mp = x2 + y2 + 1 < p2 con 0 < m < p. Podemosescribir también mp = x2 + y2 + 12 + 02, luego vemos que se ha progresadoen el teorema y habríamos terminado sim = 1. La idea ahora es de mostrarque podemos "descender m"hasta conseguir que p se escriba como sumade cuatro cuadrados, esta forma de proceder se conoce como el método deldescenso infinito de Fermat.

Sea l > 0 el número entero más pequeño tal que lp = x2 + y2 + z2 + w2.Vamos a mostrar que l = 1. Sabemos de antemano que l < p puesto quel ≤m.

Afirmación: l es impar.

En efecto, si l es par podemos suponer que x+ y, x− y, z+ w, z− w son paresya que lp = par = x2 + y2 + z2 + w2, de donde si pérdida de generalidadvemos que hay tres posibilidades:

(1) x, y deben ser pares y z, w impares.

(2) x, y, z, w son todos impares.

(3) x, y, z, w son todos pares.

39

luego

12lp =

(12

(x+ y))2

+(12

(x − y))2

+(12

(z+ w))2

+(12

(z − w))2

lo cual no es posible ya que l es minimal.

Sabemos ahora que l debe ser impar, supongamos que l > 1 y pongamosn = x′2 + y′2 + z′2 + w′2 donde x′, y′, z′,w′ son los representantes más pe-queños de x, y, z, w respectivamente módulo l. Claramente n ≡ 0 mod l yaque n ≡ x2 + y2 + z2 + w2 = lp ≡ 0 mod l, pero n > 0 ya que si n = 0⇒ x′ =y′ = z′ = w′ = 0⇒ l | x, y, z, w⇒ l2 | x2 + · · · + w2 = lp⇒ l | p lo cual no es

posible ya que p primo. Además n < 4(

12 l

)2= l2 puesto que x′, y′, z′,w′ son

los representantes más pequeños de x, y, z, w respectivamente y como l esimpar x′, y′, z′,w′ < 1

2 l.

Como l | n y n < l2 ⇒ n = kl con 0 < k < l. La fórmula (5.3) nos diceque (kl)(lp) es suma de cuatro cuadrados: (kl)(lp) = a2 + b2 + c2 + d2.

Afirmación: l2 | a2,b2, c2,d2.

En efecto, a = xx′ + yy′ + zz′ + ww′ ≡ x′2 + y′2 + z′2 + w′2 ≡ 0 mod l y demanera similar para b,c y d.

Finalmente kp =(al

)2+(bl

)2

+(cl

)2+(dl

)2

, lo que contradice de nuevo la

minimalidad de l, sigue entonces que l = 1.

5.4. Serie de ejercicios 7

1. Mostrar que las formas cuadráticas f (x,y) = x2 + y2 y g(x,y) = x2 +4xy + 5y2 son equivalentes. Deducir la “forma” de los enteros que seescriben de la forma x2 + 4xy + 5y2.

2. Encontrar dos formas cuadráticas f y g que no sean equivalentespero que representen el mismo subconjunto de enteros S ∈ Z, es decirf (Z2) = g(Z2).Indicación: Utilizar el teorema de la suma de cuatro cuadrados.

3. Sea E el conjunto de los enteros que no se escriben como suma de 3

40

cuadrados. Mostrar que

lımsupx→∞

1x|{a ≤ x : a ∈ E}| ≥ 1

8.

4. Mostrar que ningún entero de la forma 22k+1, con k ≥ 1, puede escri-birse como suma de 4 cuadrados estrictamente positivos.

41

6. Aproximaciones diofánticas

Consideremos las inclusiones N ⊂ Z ⊂ Q ⊂ R ⊂ C. Sabemos gracias anuestros primeros cursos de análisis y álgebra queQ se obtiene de Z a par-tir del cuerpo de fracciones de este último y que a su vez R es la comple-ción arquimediana de Q, finalmente añadiendo un elemento i verificandoi2 = −1 obtenemos una extensión de R que denotamos por C. Los tres pri-meros conjuntos de estas inclusiones, N,Z,Q son numerables, mientras quelos dos últimos R,C no. Si imaginamos a Q en la recta real, el hecho queéste sea numerable (¡además de ser de medida de Lebesgue nula!) y R alcontrario no lo sea, nos dice que existen agujeros entreQ y R. Para intentarcomprender qué sucede y cómo llenar esos agujeros, vamos a interesarnosen los números algebraicos.

Definición 17. Un número α ∈ C se llama algebraico si existe un polinomiono nulo P ∈Q[X] tal que P (α) = 0. Denotaremos por Q el conjunto de númerosalgebraicos sobre Q.

Observemos que en la definición, podemos exigir de manera equivalenteun polinomio no nulo P ∈ Z[X]. Llamamos polinomio minimal de un nú-mero algebraico α al único polinomio de Q[X] unitario de grado minimalque se anula en α. Sea d el grado del polinomio minimal de α. Diremosque α es un número algebraico de grado d.

Diremos que α ∈ C es trascendente si α no es algebraico. A finales delsiglo XIX se demostró que e (Hermite) y π (Lindeman) son trascendentes.

Determinar si un número es o no transcendente es en general muy difí-cil, de hecho aún no se ha podido demostrar si por ejemplo e +π es o notrascendente, así como también: πe, ee,ππ,λ = lımN→∞

∑Nn=1

1n − logN que

aún son preguntas abiertas.

6.1. Teorema de Dirichlet

Teorema 11. (Dirichlet) Para todo θ ∈ R et para todo Q > 1, existen p,q ∈ Z,0 < q < Q tales que |qθ − p| ≤ 1

Q .

Demostración. Sea θ ∈ R, Q > 1. Supongamos que Q ∈ Z y consideremoslos Q+ 1 números siguientes

0, {θ}, {2θ}, · · · , {(Q − 1)θ},1 ∈ [0,1]

42

Dividamos el intervalo [0,1] en Q sub-intervalos de largo 1Q . Por el princi-

pio de Dirichlet, existen dos de esos números x1 y x2 que se encuentran enel mismo sub-intervalo.

Si x1,x2 , 1, existen Q − 1 ≥ m1 > m2 ≥ 0 tales que |{m1θ} − {m2θ}| ≤ 1Q .

Pongamos q =m1 −m2 y p = [m1θ]− [m2θ], tenemos que 0 < q < Q. Luego

|pθ − q| = |(m1 −m2)θ − ([m1θ]− [m2θ])| = |{m1θ} − {m2θ}| ≤1Q

Si uno de los xi (i = 1 o i = 2) es igual a 1, se procede de la misma manera(en ese caso para uno de los xi no será necesario escribir partes enteras ypartes fraccionarias).

Si Q no es entero, procedemos como antes con [Q] + 1 y luego observa-mos que 1

[Q]+1 <1Q .

Observación 9. Podemos exigir que (p,q) = 1, ya que si (p,q) = d > 1 pode-mos poner en evidencia el factor d del miembro izquierdo de la desigual-dad, luego dividir por d ambos lados y finalmente observar que 1

dQ <1Q .

Corolario 4. (1) Si θ es irracional, entonces existe una infinidad de racio-nales p

q tales que |θ − pq | ≤1q2 , con (p,q) = 1.

(2) Si θ es racional, entonces existe un número finito de racionales pq tales

que |θ − pq | ≤1q2 , con (p,q) = 1.

Este corolario puede parecer contra-intuitivo ya que nos dice que los ra-cionales no se pueden aproximar tan bien como los irracionales con suce-siones de racionales (salvo la sucesión constante igual al número racionalen cuestión). Este fenómeno sucede por algo que ya habíamos mencionadoantes, que es el hecho que Q visto sobre R está lleno de agujeros.

Demostración. Para no preocuparnos de problemas de signo, vamos a su-poner θ ≥ 0. Esto no supone ninguna restricción ya que para θ < 0 tendre-mos demostrado el corolario para −θ y un simple cambio en el signo de pnos llevará de vuelta al resultado que buscamos.

(1) Sea θ irracional y Q > 1. Por el teorema 11 existe pq tal que |θ − pq | ≤

1q2

con (p,q) = 1. Como θ no es racional, 0 , |θ − pq | ≤ 1, luego existe Q2 > 1

tal que Q2 >1

|qθ−p| . De nuevo por el teorema 11 existe p′

q′ tal que |q′θ −p′ | <

43

1Q2< |qθ − p| (y si se quiere con (p′,q′) = 1) de donde p

q ,p′

q′ y así sucesiva-mente.

(2) Si θ es racional, entonces θ = ab con a,b enteros. Para p

q ,ab , tenemos

|θ − pq | = |aq−pbqb | ≥

1qb ya que |aq − pb| ≥ 1 puesto que es un entero no nulo.

Ahora bien, si q > b entonces 1bq >

1q2 , luego q ≤ b lo que nos da una co-

ta superior para q y por lo tanto un número finito de pq que satisfacen la

propriedad que buscamos.

Este resultado es optimal en el sentido siguiente:

Teorema 12. Si θ es algebraico, entonces ∀ε > 0 ∃c > 0 tal que∣∣∣∣∣θ − pq

∣∣∣∣∣ ≥c

q2+ε ∀p

q.

Un método para encontrar buenas aproximaciones racionales es el de lasfracciones continuas.

6.2. Fracciones continuas

El principio de base de las fracciones continuas es el de establecer bi-yecciones entre los irracionales y las sucesiones de a0, a1, a2, ... con ai ∈Z, ai > 0 si i , 0 y los racionales y las sucesiones finitas a0, a1, a2, ..., an conai ∈ Z, ai > 0 si i , 0.

Sea θ ∈ R, pongamos a0 = [θ]. Si a0 , θ, escribiremos θ = a0 +1θ1

, θ1 > 1 y

luego pondremos a1 = [θ1]. Si a1 , θ1, escribiremos θ = a0 +1

a1 +1θ2

, θ2 > 1

y luego pondremos a2 = [θ2] y así sucesivamente.

Si el algoritmo se termina en un número finito de etapas, entonces exis-te un n tal que an = θn y tenemos

θ = a0 +1

a1 +1

a2+. . . +1an

44

Notación:a0 +

1

a1 +1

a2+. . . +1an

= [a0, · · · , an]

Si el algoritmo no se termina, obtenemos una sucesión a0, a1, a2, · · · cona0 ∈ Z, ai ∈ N∗ si i ≥ 1. Más adelante mostraremos que en este caso θ esirracional.

Ejemplos

(1)18735

= [5,2,1,11]

(2) π = [3,7,15,1,292,1,1,2, · · · ]

(3) ϕ = [1,1,1, · · · ] donde ϕ es el número de oro, el cual satisface la ecua-ción x2 − x − 1 = 0.

El siguiente teorema nos permitirá justificar (por ejemplo) las escriturasϕ = [1,1,1, · · · ],π = [3,7,15,1,292,1,1,2, · · · ], etc.

Teorema 13. Si an , θn ∀ n, la sucesión [a0, a1, · · · ] converge hacia θ cuando

n → ∞. Además sipnqn

= [a0, a1, · · · , an] con (pn,qn) = 1, entonces∣∣∣∣∣θ − pnqn

∣∣∣∣∣ ≤1

qnqn+1.

Definición 18. Los racionalespnqn

se llaman los convergentes de θ.

Corolario 5. Si θ es racional, el algoritmo de escritura en fracción continua setermina.

Demostración. Si θ =ab

, entonces por el teorema 13 tenemos que | bθqn − bpn︸ ︷︷ ︸∈Z

|≤

bqn

n→∞−→ 0 y por lo tanto debe existir un n tal que θ =

pnqn

= [a0, · · · , an]

Regresemos ahora el teorema 13:

Demostración. Definimos por recurrencia las sucesiones siguientes: pn =anpn−1 + pn−2 con p0 = a0, p1 = a0a1 + 1 y qn = anqn−1 + qn−2 con q0 = 1,q1 = a1.

45

Afirmación 1: ∀x > 0, [a0, · · · , an−1,x] =xpn−1 + pn−2

xqn−1 + qn−2si n ≥ 2.

En efecto, procedamos por inducción sobre n ≥ 2: [a0, a1,x] =xp1 + p0

xq1 + p0,

supongamos ahora cierta la afirmación 1 para n ≥ 2 y mostrémosla paran+ 1

[a0, · · · , an,x] = a0 +1

a1+. . . + an−1 +1

an +1x

= {y = an +1x} = [a0, · · · , an−1, y]

luego por hipótesis de inducción

[a0, · · · , an−1, y] =ypn−1 + pn−2

yqn−1 + qn−2=

(xan+1x

)pn−1 + pn−2(

xan+1x

)qn−1 + qn−2

=

=x

pn︷ ︸︸ ︷(anpn−1 + pn−2)+pn−1

x (anqn−1 + qn−2)︸ ︷︷ ︸qn

+qn−1

luego poniendo x = an en la afirmación se tiene [a0, · · · , an] =anpn−1 + pn−2

anqn−1 + qn−2=

pnqn

.

Afirmación 2: ∀n ≥ 0, pnqn+1 − pn+1qn = (−1)n+1

Como en la afirmación 1, se procede por inducción sobre n.

Observemos que la afirmación 1 nos dice que (pn,qn) = 1 ∀n, es decir

acabamos de mostrar que lospnqn

son los convergentes de θ.

La afirmación 2 también implica que∣∣∣∣∣pnqn − pn+1

qn+1

∣∣∣∣∣ =1

qnqn+1.

Observación:p2k

q2k≤ θ,

p2k+1

q2k+1≥ θ (se muestra también por inducción).

La observación implica en particular que∣∣∣∣∣pnqn −θ

∣∣∣∣∣ ≤ 1qnqn+1

.

46

6.3. Serie de ejercicios 8

1. Suponiendo que 3.1415926 es una aproximación correcta de π dedecimales, calcular los 4 primeros convergentes de π.

2. Sea θ = [a0, ..., an] con a0 > 0. Mostrar que 1θ = [0, a0, ..., an].

3. Determinar la fracción continua de√n2 + 1 para todo n ≥ 1.

4. Sean θ1 y θ2 dos raíces del polinomio x2 − ax − 1 con a ∈ N∗ y θ1 >0. Mostrar que los denominadores de los convergentes de θ1 estándados por

qn−1 =θn1 −θ

n2

θ1 −θ2.

Deducir la fórmula de n-ésimo número de Fibonacci.

6.4. Teorema de Liouville

De nuestros primeros cursos de análisis sabemos que los números ra-cionales son densos en los reales, esto quiere decir que todo número reales el límite de una sucesión de números racionales. Hemos visto que unaforma eficaz de aproximar un número real por una sucesión de númerosracionales es la de utilizar fracciones continuas. Recordemos que el teore-ma de Dirichlet (teorema 11) nos dice que si α ∈ R \Q entonces existe una

infinidad de racionalesp

qcon (p,q) = 1 tales que

∣∣∣∣∣α − pq∣∣∣∣∣ ≤ 1

q2 . Como sabe-

mos, esto no es cierto si α es racional. Tal vez sea más interesante aproxi-mar números algebraicos. Desgraciadamente el siguiente teorema nos diceque tales números no se aproximan bien con números racionales.

Teorema 14. (Liouville 1840) Sea α ∈ R un número algebraico de grado d

mayor o igual a 2. Existe una constante c = c(α) tal que∣∣∣∣∣α − pq

∣∣∣∣∣ ≥ c

qdpara todo

racionalp

q.

Demostración. Sean pq un número racional cualquiera, P polinomio mini-

mal de α (entonces deg P = d). Notemos que P (pq ) , 0, si no dividimos P

por (X− pq ) y eliminamos los denominadores del polinomio resultante paraobtener Q ∈ Z[X] con Q(α) = 0 y deg Q = deg P − 1, lo cual no es posibleya que P es el polinomio minimal de α.

47

Tenemos que 0 , qdP (pq ) ∈ Z y entonces∣∣∣∣qdP (

pq

)∣∣∣∣ ≥ 1 luego∣∣∣∣∣∣P(p

q

)∣∣∣∣∣∣ ≥ q−d (4)

Por otra parte, el teorema de los valores intermediarios se tiene

p(α)− p(p

q

)=

(α −

p

q

)P ′(ζ)

para un cierto ζ ∈ [α, pq ] o ζ ∈ [pq ,α]. Pongamos c(α)−1 := maxα−1≤x≤α+1

|P ′(x)|(observe que c(α) , 0 ya que al menos P ′(ζ) , 0). Finalmente la ecuación(4) y el hecho que P (α) = 0 nos dan∣∣∣∣∣α − pq

∣∣∣∣∣ ≥ c(α)qd

Una generalización de este resultado es el teorema de Roth (que no de-mostraremos acá):Sea α un numero irracional algebraico, ∀ε > 0 ∃c = c(α,ε) tal que ∀ p ∈ Z

∀ q ∈ N∗∣∣∣∣∣α − pq

∣∣∣∣∣ ≥ c

q2+ε .

El teorema de Roth muestra que la medida de irracionabilidad de un nú-mero algebraico es 2, éste teorema y el de Liouville permiten demostrar,por contraposición, la irracionalidad de muchos números como veremos acontinuación.

6.5. Números trascendentales

En esta sección mostraremos que existe una infinidad de números trascen-dentales.

Proposición 13. Sea b > 1 entero y (an)n≥1 una sucesión de enteros compren-didos entre 0 y b − 1, el número real

O :=∞∑n=1

anbn!

es trascendente.

48

Demostración. El criterio del cociente nos garantiza la convergencia de O.

Pongamos qn = bn! y pn = qnn∑k=1

akbk!

, entonces

∣∣∣∣∣O − pnqn∣∣∣∣∣ =

∞∑k=n+1

akbk!≤

∞∑k=n+1

b − 1bk!

<∞∑

k=(n+1)!

b − 1bk

=1

b(n+1)!−1≤ 1qnn

Ahora si O fuese algebraico de grado d, por el teorema de Liouville (teo-rema 14) debe existir una constante c tal que para todo racional p

q , O setenga ∣∣∣∣∣O − pq

∣∣∣∣∣ ≥ c

qd

Pero entoncesc

qdn≤

∣∣∣∣∣O − pnqn∣∣∣∣∣ < 1

qnn

lo cual implica que c ≤ qd−nn para todo n, es decir la relación sólo es posiblesi c = 0, lo que contradice el teorema 14 ya que c > 0.

Cantor demostró en 1874 que R es no numerable y que el conjunto denúmeros algebraicos Q es numerable. Con la proposición precedente aca-bamos de construir una infinidad de números trascendentales non nume-rables, exactamente 2ℵ0 , llamados trascendentes de Liouville. Desde luego,no son los únicos:

Teorema 15. (Hermite 1873) El número e :=∞∑n=1

1n!

es trascendente.

Demostración. Etapa 1: Sea I(t) =∫ t

0et−xf (x)dx donde f es un polinomio

a coeficientes reales de grado deg f =m. Integrando por partes

I(t) =[−et−xf (x)

]t0−∫ t

0−et−xf ′(x)dx = −f (t) + etf (0) +

∫ t

0et−xf ′(x)dx =

= · · · iterar · · · = etm∑j=0

f (j)(0)−m∑j=0

f (j)(t)

Etapa 2:

|I(t)| ≤∫ t

0

∣∣∣et−x∣∣∣ |f (x)|dx =∫ t

0et−x︸︷︷︸≤et

(|am| |x|m︸︷︷︸≤tm

+ · · ·+ |a0|)dx ≤ tet f (t)

49

donde f (x) = |am|xm + · · ·+ |a0|

Etapa 3: Supongamos que e es algebraico de grado n, existe entonces unarelación a0 + a1e+ · · ·+ anen = 0 con a0 , 0, aj ∈ Z.Sea f (x) = xp−1(x−1)p(x−2)p · · · (x−n)p con p primo grande. El grado de fesm = deg f = pn+p−1. Podemos escribir f (x) = g(x)(x−k)p k = 1,2, · · · ,n.Derivando j veces tenemos

f (j) =j∑i=0

(ji

)g(j−i)(x)

di

dxi(x − k)p

de donde f (j)(k) = 0 si j < p y f (j)(k) =(jp

)g(j−p(k)p! si j ≥ p.

Etapa 4: Sea J = a0I(0) + a1I(1) + · · ·+ anI(n), por la etapa 1 se tiene

J = a0

e0m∑j=0

f (j)(0)−m∑j=0

f (j)(0)

+ · · ·+ an

en m∑j=0

f (j)(0)−m∑j=0

f (j)(n)

luego por la etapa 2

J = −m∑j=0

n∑k=0

akf(j)(k)

Etapa 5: Para todo j, f (j)(k) es un entero divisible por p! con k = 1, · · · ,n.

Para j < p − 1 tenemos f (j)(0) = 0 y si j ≥ p − 1 se tiene f (j)(0) =(j

p − 1

)(p −

1)!h(j−p+1)(0) donde h(x) =f (x)xp−1 . Sigue entonces que (p − 1)! | f (p−1)(0) pero

p - f (p−1)(0)

Etapa 6: Elijamos ahora p primo tal que p > a0, · · · , an de forma que p -a1, · · · , a2.

J = −a0f(p−1)(0)− a0

m∑j=p

f (j)(0)−m∑i=p

n∑k=1

akf(j)(k)

Ahora bien, p divide todos los términos de la expresión de arriba exceptoel primero, sigue entonces que J , 0. Como (p − 1)! divide todos los térmi-nos de J no nulo, se tiene que |J | ≥ (p − 1)!

50

Etapa 7: Para f (x) = xp−1(x − 1)p · · · (x −n)p, si k ≤ n se obtiene

f (k) ≤ (2n)m

donde m = np+ p − 1 ≤ 2np. Luego

|J | ≤etapa 2

|a1|ef (1) + · · ·+ |an|nenf (n) ≤n∑k=0

|ak |kek︸ ︷︷ ︸:=D(n)

(2n)2np

Etapa 8: Finalmente hemos mostrado que

(p − 1)! ≤ |J | ≤D(n)(2n)2np

luego enviando p→∞ llegamos a una contradicción ya que p! crece másrápido que cp.

6.6. Serie de ejercicios 9

1. Determinar el número racional cuya fracción continua es [1,2,3,4].

2. Mostrar que, para todo θ irracional, los denominadores de los con-

vergentes de θ satisfacen la desigualdad qn ≥(1+√

5)n−1

2 . Además, mos-

trar que si an ≤ A para todo n, entonces qn ≤(A+√A2+42

)n.

3. Sea θ ∈ R. Supongamos que en la fracción continua que representa aθ, la sucesión an sea creciente. Mostrar que si an+1 > (an + 1)n

2para

todo n, entonces θ es trascendente.Indicación: Utilizar el hecho que

∣∣∣∣θ − pnqn ∣∣∣∣ < 1an+1q

2n.

4. Admitamos el teorema de Roth. Mostrar que la suma a−b+a−b2+a−b

3+

· · · es trascendente para todos los enteros a ≥ 2,b ≥ 3.Teorema de Roth: Para todo irracional algebraico α y todo ε > 0, existeuna constante c = c(α,ε) tal que para todo racional pq , se tiene∣∣∣∣∣α − pq

∣∣∣∣∣ > c

q2+ε .

51

A continuación presentamos un teorema (sin demostración) que nos daráun nuevo criterio de trascendencia.

Teorema 16. (Lindemann 1882) Sean α0, · · · ,αn números algebraicos dis-tintos y β0, · · · ,βn números algebraicos no todos nulos. Entonces β0e

α0 + · · · +βne

αn , 0.

Corolario 6. Los números siguientes son trascendentes: eα, π, logα, sinα,cosα, tanα con α algebraico α , 0,1.

No vamos a probar todo el corolario, sin embargo estudiaremos los ca-sos eα y π:

Supongamos eα algebraico, entonces existe una relación β0zn + β1z

n−1 +· · ·+ βn = 0 con βi ∈ Z no todos nulos y z = eα. De nuestros cursos de álge-bra sabemos queQ(α) es un cuerpo, en particular para k = 1, · · · ,n tenemosque αk = kα es algebraico, lo que contradice el teorema 16.

Para ver que π es trascendente (un resultado muy importante que com-binado con la teoría de Galois puso fin al problema de la cuadratura delcírculo) observemos que (−1) · e0 + 1 · e2πi = 0 y de nuevo si suponemos πalgebraico se contradice el teorema 16.

Recordemos que el teorema de Roth nos dice que si α es un número alge-braico no racional, entonces para todo ε > 0 existe una constante c = c(α)

tal que para todo racionalp

qse tiene

∣∣∣∣∣α − pq∣∣∣∣∣ ≥ c

q2+ε . Este teorema nos per-

mitirá mostrar el siguiente resultado concerniente a las soluciones enterasde ciertos tipos de ecuaciones polinomiales homogéneas en dos variables.

Teorema 17. (Thue 1909) Sea F(x,y) = a0xn+a1x

n−1y + · · ·+anyn una formairreducible de grado n ≥ 3, ai ∈ Z y sea m ∈ Z. La ecuación F(x,y) = m tiene alo sumo un número finito de soluciones (x,y) ∈ Z2.

Demostración. Por irreductibilidad de F podemos escribir

a0(x −α1y) · · · (x −αny) =m

con todos los αi algebraicos no racionales diferentes.

Supongamos por el absurdo que la ecuación F(x,y) = m posee una infini-dad de soluciones, por el rol simétrico que tienen x e y podemos suponersin pérdida de generalidad de hay una infinidad de y tales que y → ∞.

52

Pongamos ε := mini,j

d(αi ,αj) y dividamos la expresión de arriba por y (que

podemos suponer non nulo), tenemos(xy−α1

)· · ·

(xy−αn

)=

ma0yn

Como y puede ser elegido arbitrariamente grande, uno de los factores a la

izquierda entre paréntesis debe ser menor aε2

, digamos que∣∣∣∣∣xy −α1

∣∣∣∣∣ ≤ ε2.

Debido a la elección de ε, para el resto se tiene∣∣∣∣∣xy −αi

∣∣∣∣∣ > ε2

si i , 1, de

donde ∣∣∣∣∣xy −α1

∣∣∣∣∣ ≤ C|y|n

donde C = C(ε,m,a0). Pero según el teorema de Roth, esto no es posible.

En particular este teorema nos dice que x3−2y3 = 11 posee un númerofinito de soluciones, algo que no es trivial de ver si uno no conoce esteteorema. La hipótesis sobre el grado de la forma es importante, he aquí uncontra-ejemplo:

Definición 19. Sea d un entero positivo sin factores cuadrados. Las ecuacionesde la forma x2 − dy2 = 0 se llaman ecuaciones de Pell.

Más adelante veremos una demostración sobre la infinitud de solucio-nes de una ecuación de Pell.

Un fenómeno similar al teorema de Thu ocurre con las ecuaciones de tipoxn + yn = zn de las cuales buscamos todas las soluciones enteras. Sabemosque para n = 2 las soluciones a esta ecuación se conocían desde hace tiem-po, son la llamadas ternas pitagóricas: x =m2−n2, y = 2mn,z =m2 +n2 paratodos m,n ∈ Z. Por el contrario, el caso n ≥ 3 conocido como último teore-ma de Fermat no posee más que la solución entera trivial, este teorema fuedemostrado en 1994 por Andrew Willes.

Proposición 14. Sea x una solución de xm + c1xm−1 + · · · + cm = 0 ci ∈ Z,

entonces o bien x ∈ Z o bien x es irracional.

Demostración. Supongamos que x = ab con (a,b) = 1, tenemos que am +

c1am−1b + · · · + cmbm = 0 ⇒ b | am, pero (a,b) = 1, luego b = 1 y entonces

x es un entero.

53

Por ejemplo esta proposición generaliza la posiblemente famosa demos-tración del hecho que

√2 es irracional.

Sabemos por el teorema 16 que e es trascendente y por lo tanto irracio-nal. Sin embargo, no necesitamos de invocar al teorema de Lindemannpara probar la irracionalidad de e

Proposición 15. e es irracional.

Demostración. Supongamos que e = ab . Para k > b tenemos que b | k! y en-

toncesα := k!

(e − 1− 1

1!− 1

2!− · · · − 1

k!

)∈ Z

Pero

0 < α =1

k + 1+

1(k + 1)(k + 2)

+ · · · ≤ 1(k + 1)

+1

(k + 1)2 + · · · = 1k< 1

lo cual no es posible ya que α es un entero y por lo tanto no puede estarestrictamente comprendido entre 0 y 1.

A continuación enunciaremos otro teorema de Lindemann sobre latrascendencia de π, la cual no demostraremos en este curso.

Teorema 18. (Lindemann 1877) π es trascendente.

Demostración. Omitida.

Corolario 7. La cuadratura del círculo es imposible.

Demostración. Ver curso de Álgebra de segundo año.

Para no irnos con las manos vacías, vamos a demostrar que π es irracional.Este hecho fue probado por Lambert en 1750. La prueba que presentare-mos es más corta y no corresponde a la original.

Teorema 19. π es irracional.

Demostración. Sea fn(x) =xn(1− x)n

n!=

1n!

2n∑m=n

cmxm cm ∈ Z. Para 0 < x < 1

se tiene que 0 < fn(x) < 1n! . Observemos también que f (x) = f (1− x) luego

f(m)n (1) = (−1)mf (m)

n (0) =

0 m < n

m!n! cm n ≤m ≤ 2n

0 m > 2n

54

en decir f (m)n (0), f (m)

n (1) ∈ Z. Supongamos que π2 = ab , sea

G(x) = bn(π2nf (x)−π2n−2f (2)(x) + · · ·+ (−1)nf (2n)(x)

)entonces para f de la forma de cualquier fj(x) se ve de inmediato queG(0),G(1) ∈ Z.

Por otra parte

ddx

(G′(x)sinπx −πG(x)cosπx) = (G′′(x) +π2G(x))sinπx =

= bn(π2nf ′′(x) + · · ·+ (−1)2f 2n+2(x) +π2n+2f (x) + · · ·+ (−1)π2f (2n)(x))sinπx

= bn(π2n+2f (x))sinπx = π2anf (x)sinπx

entonces

π

∫ 1

0an sinπxf (x)dx =

[G′(x)sinπx −πG(x)cosπx]10 = G(0) +G(1) ∈ Z

pero

0 < π∫ 1

0an sinπx︸︷︷︸∈(0,1)

f (x)︸︷︷︸0<f (x)< 1

n!

<πan

n!< 1

si n es suficientemente grande, lo cual no es posible.

55

7. Cuerpos cuadráticos

7.1. Cuerpo de números

Un cuerpo de números K es una extensión finita de Q. En esta secciónabordaremos de una manera simplificada ciertos aspectos de la teoría decuerpos estudiada en cursos de álgebra de segundo año. Recordemos queun número α ∈ C es algebraico si existe p ∈ Q[X] tal que p(α) = 0. El con-junto de números algebraicos es denotado por Q.

El polinomio minimal P de α es el polinomio de grado minimal que seanula en α y tiene como coeficiente principal (coeficiente del término demás alto grado) igual a 1. Como las otras raíces de P comparten el mis-mo polinomio minimal que α, a menudo se las llama conjugadas de α ypodemos notarlas por α1 = α, · · · ,αd donde d = deg P .

Definición 20. Sean α un número algebraico y α1, · · · ,αd las conjugadas desu polinomio minimal, llamamos norma de α a la expresión:

N (α) = α1 · · ·αd

y Traza de α a:T (α) = α1 + · · ·+αd

Observemos que si P (X) = Xd + a1Xd−1 + · · ·+ ad es el polinomio mini-

mal de α, tanto N (α) como T (α) son racionales ya que N (α) = (−1)dad yT (α) = −a1.

Definimos Q(α) := {p(α) | p ∈Q[X]} = {b0 + b1α + · · ·+ bd−1αd−1,bi ∈Q}.

Observaciones:

1. La segunda igualdad viene del algoritmo de la división sobre el anillode polinomios Q[X]: ∀p ∈ Q[X], p(X) = a(X)P (X) + r(X) con deg r ≤d − 1 y P polinomio minimal de α, luego evaluando en α se tienep(α) = r(α) = b0 + · · ·+ bd−1α

d−1.

2. En un contexto más general, el conjunto Q(α) se define como el máspequeño cuerpo intermediario entre Q y una extensión de Q conte-niendo α.

De nuestra definición, vemos inmediatamente que Q(α) es un Q-espaciovectorial de dimensión d con base {1,α,α2, ...,αd−1} (para ver que es una

56

familia Q-linealmente independiente utilizar el hecho que el grado delpolinomio minimal de α es d). Decimos entonces que el grado de Q(α) so-bre Q es d y escribimos [Q(α) :Q] = d.

Nuestra definición nos dice también que Q(α) es un anillo, pero no que setrata de un cuerpo. Sean entonces 0 , q ∈ Q(α) y P el polinomio minimalde α con deg P = d, tenemos que q y P son relativamente primos (como po-linomios en X) ya que deg q ≤ d−1 y P es irreductible (si no P no sería mi-nimal), entonces por el algoritmo de la división euclidiana existen polino-mios a,b ∈Q[X] con deg a,deg b ≤ d −1 tales que a(X)q(X) + b(X)P (X) = 1,luego evaluando en α se tiene a(α)q(α) = 1. Q(α) es entonces un cuerpo.

Ejemplos

(1) α = i =√−1

Q(i) se llama cuerpo Gaussiano. El polinomio minimal de i es P (X) =X2 + 1, las conjugadas de i son entonces α1 = α = i y α2 = −i. LuegoN (i) = i · (−i) = 1, T (i) = i + (−i) = 0.

(2) α =√

2El polinomio minimal de

√2 es P (X) = X2 − 2, sus conjugadas son

α = α1 =√

2 y α2 = −√

2, luegoN (√

2) = −2, T (√

2) = 0.

Definición 21. Un número algebraico se llama entero algebraico si su poli-nomio minimal P ∈ Z[X].

Observemos que por definición el polinomio minimal P de un entero alge-braico es unitario, es decir el coeficiente del término de más alto grado esigual a 1. Por otro lado, si α es un entero algebraico sigue inmediatamentequeN (α),T (α) ∈ Z (simplemente comparar con los coeficientes de P ).

Es posible mostrar que los enteros algebraicos forman un anillo, a me-nudo denotado por O. Para K ⊃ Q una extensión finita de Q (es decir uncuerpo de números), definimos por OK := O∩K el conjunto de los enterosalgebraicos de K . Este conjunto es también un anillo llamado anillo de losenteros algebraicos de K .

Definición 22. Un entero algebraico (sobre K) ε , 0 es una unidad si1ε∈ OK .

Ejemplo Si tomamos K = Q, entonces OQ = Z, es decir los enteros al-gebraicos de Q son los enteros que conocemos. En efecto, si α ∈ OQ existeuna relación de tipo αd + a1α

d−1 + · · · + ad = 0 con los ai ∈ Z, luego por la

57

proposición 14 o bien α ∈ Z o bien α es racional, en el segundo caso el po-linomio minimal de α es un monomio unitario, entonces α tiene que serentero. Tenemos entonces que las unidades de OQ son {−1,1}.

Definición 23. Sean α,β,γ ∈ OK

(1) Decimos que α es divisible por 0 , β siαβ∈ OK .

(2) Decimos que α y β están asociados si existe ε ∈ OK unidad tal que α = εβ.

(3) Decimos que α es irreducible si para todo β tal que β | α, entonces β estáasociado a α o bien β es una unidad.

(4) Decimos que α es primo si para todos β y γ tales que α | βγ , entoncesα | β o bien α | γ .

7.2. Cuerpos cuadráticos

Sea 0,1 , d ∈ Z sin factores cuadrados, un cuerpo cuadrático es unaextensión algebraica de Q de grado 2. Se puede mostrar fácilmente que laasociación d 7→ Q(

√d) nos da una biyección entre los cuerpos cuadráticos

y tales d. Entonces de forma más explicita un cuerpo cuadrático es uncuerpo de la forma

Q(√d) = {u + v

√d | u,v ∈Q}

con d , 0,1 sin factores cuadrados.

A continuación vamos a determinar los enteros algebraicos de Q(√d)

Proposición 16. dado un cuerpo de números K = Q(√d) con d , 0,1 sin

factores cuadrados, el anillo de enteros algebraicos de K es un anillo de la formaZ[ω] = {m+nω |m,n ∈ Z} donde

ω =

√d si d ≡ 2,3 mod 4

1 +√d

2si d ≡ 1 mod 4

Demostración. Sea α = u + v√d un entero algebraico. Pongamos a = 2u y

b = 2v. El polinomio minimal de α es P (X) = X2 − aX + c, en efecto seaα = u−v

√d el conjugado de α, tenemos que α+ α = 2u = a y c = αα. Como

α es un entero sobre K , se sigue que a,c ∈ Z.

58

4c = 4αα = 4N (α) = 4(u + v√d)(u − v

√d) = a2 − db2 ⇒ a2 − 4c = db2 ∈ Z,

ahora si b = mn tendríamos que m2

n2 d ∈ Z, pero d es sin factores cuadrados,luego n = ±1 de donde b ∈ Z.Tenemos entonces que a,b y c son enteros, nos resta estudiar las posibili-dades para d:

Caso 1: Si d ≡ 2,3 mod 4

4c = a2 − db2 mod 4=⇒ a2 ≡ 2b2 mod 4 o a2 ≡ 3b2 mod 4. Ahora bien, como

x2 ≡ 0,1 mod 4 para todo x ∈ Z, las dos ecuaciones modulares de arriba noson posibles a menos que a2 ≡ b2 ≡ 0 mod 4, luego a y b deben ser pares yentonces u,v ∈ Z. Se tiene entonces que OQ(

√d) = {m+n

√d |m,n ∈ Z}.

Caso 2: Si d ≡ 1 mod 4

4c = a2 − db2 mod 4=⇒ a2 ≡ b2 mod 4⇒ a ≡ b mod 2, es decir a y b son ambos

pares o impares, en todo caso a−b2 ∈ Z, de donde u − v ∈ Z. Luego podemos

escribir

u + v√d = (u − v) + v(1 +

√d) = u − v︸︷︷︸

∈Z

+b1 +√d

2

de este modo los enteros algebraicos sobre Q(√d) son engendrados sobre

Z por {1, 1+√d

2 }. Finalmente OQ(√d) =

{m+n1+

√d

2 |m,n ∈ Z}

Ejemplos

(1) −1 ≡ 3(4), luego por la proposición 16 tenemos que el anillo de ente-ros Gaussianos es Z[i] = {m+n

√d |m,n ∈ Z}.

(2) 5 ≡ 1(4), luego el anillo de enteros algebraicos deQ(√

5) es Z[

1+√

52

]={

m+n1+√

52 |m,n ∈ Z

}.

7.3. Unidades

En el contexto de un cuerpo cuadrático, es fácil verificar que la normade un número algebraico α es multiplicativa. En efecto, si α = u + v

√d, su

conjugado es α = u −v√d, luego para otro número algebraico β un cálculo

elemental daN (αβ) =N (α)N (β).

59

Proposición 17. Sea ε un entero algebraico, ε es una unidad si y sólo siN (ε) =±1.

Demostración. ⇐) Si ε es una unidad, por definición 1ε es algebraico, luego

ε · 1ε = 1⇒ N (ε)N (1

ε ) = N (1) = 1, pero N (ε),N (1ε ) ∈ Z, luego necesaria-

menteN (ε) = ±1.

⇒) Sea ε un entero algebraico tal que N (ε) = εε = ±1 la afirmación siguetrivialmente.

Para un cuerpo cuadrático Q(√d) es fácil demostrar que si d < 0, y

d ≡ 2,3(4) el número de unidades es finito. En efecto hay que estudiarlos valores enteros de la ecuación x2 − dy2 = ±1, descartamos la ecuaciónx2−dy2 = −1 ya que su única solución entera es (x,y) = (0,0), luego si con-sideramos la ecuación x2 − dy2 = 1 es claro que esta ecuación no puedetener más que un número finito de soluciones enteras. Si d > 0 el númerode unidades es infinito, éste hecho es una consecuencia directa del teoremade Dirichlet de las unidades (que no demostraremos acá ya que correspondea una rama de la teoría de números que no está al alcance de este cursointroductorio: la teoría algebraica de números).

Antes de terminar esta sección examinemos un ejemplo interesante:

Consideremos el cuerpo cuadrático K = Q(√−5). En el anillo de enteros

de este cuerpo se tiene la factorización

6 = 2 · 3 = (1 +√−5)(1−

√−5).

Vamos a mostrar que la factorización de 6 en producto de irreducibles noes única (única en el sentido “módulo una unidad”). En efecto, la escri-tura de arriba no es suficiente para concluir, ya que a priori los factores2,3,1 +

√−5,1−

√−5 podrían estar asociados. Como −5 < 0 y −5 ≡ 3(4) te-

nemos que las unidades de OK son {1,−1}.

Mostraremos que 2 es irreducible (para 3 y 1 ±√−5 el razonamiento es

similar). Si 2 = αβ entonces 4 =N (α)︸︷︷︸∈Z

N (β)︸︷︷︸∈Z

luego los valores posibles para

N (α) son ±4,±2,±1, pero como N (α) = x2 + 5y2 los valores ±2 no puedenser posibles, en consecuencia α es una unidad o bien β es una unidad.

Podemos aprovechar también para mostrar que 2 no es primo. En efec-to 2 | 6 = (1 +

√−5)(1 −

√−5), entonces si 2 fuese primo el debe dividir

60

(1+√−5) o (1−

√−5), luego debe existir γ ∈ Z[

√−5] tal que 1±

√−5 = 2γ ⇒

6 =N (1±√−5) =N (2)N (γ) = 4N (γ)︸︷︷︸

∈Z

lo cual no es posible.

Recordemos que una anillo es factorial si todos sus elementos puedenescribirse de manera esencialmente única como producto de factores irre-ducibles. Luego el anillo de enteros algebraicos de Q(

√−5) no es factorial.

Proposición 18. OK es un anillo factorial si y sólo si todo elemento irreduciblede OK es también primo.

Recordemos que en un anillo íntegro, todo elemento primo es irreduci-ble. El anillo de enteros algebraicos de un cuerpo de números cuadráticosK = Q(

√d) es un anillo factorial si y sólo si d = −1,−2,−3,−7,−11,−15,

−43,−67,−163. Este resultado es más difícil de establecer (con teoría al-gebraica de números se puede mostrar gracias a la finitud del número declases de ideales hK que para tales d, OK es principal y por lo tanto facto-rial). Si d > 0 no se conoce muy bien lo que pasa, de hecho no se sabe sihay o no una infinitud de anillos factoriales.

7.4. Serie de ejercicios 10

1. Encontrar una infinidad de unidades diferentes en los anillos de en-teros algebraicos de Q(

√2) y Q(

√3).

2. Mostrar que la ecuación 2 · 3 = −√−6√−6 implica que el anillo de

enteros algebraicos de Q(√−6) no es factorial.

3. Explicar por qué la ecuación 2 · 11 = (5 +√

3)(5 −√

3) no constituyeuna contradicción con el hecho que el anillo de enteros algebraicosde Q(

√3) es factorial.

4. Mostrar que el anillo de enteros algebraicos de Q(√

10) no es facto-rial.Indicación: Considerar el entero algebraico 4 +

√10.

7.5. El cuerpo Gaussiano Q(i)

Definición 24. Un cuerpo cuadrático K = Q(√d) es euclidiano si su anillo

de enteros algebraicos OK posee una división euclidiana, es decir

∀α,β ∈ OK ,β , 0 ∃γ,δ ∈ OK tal que α = γβ + δ, |N (δ)| < |N (β)|

61

La existencia de la división euclidiana en un anillo implica la existenciade un máximo común divisor (módulo una unidad), además de una relaciónde Bézout:

∀α,β ∈ OK , ∃λ,µ ∈ OK tal que αλ+ βµ = (α,β) := d

Además si d es una unidad, podemos elegir λ′,µ′ ∈ OK tales que αλ′+βµ′ =1.

Proposición 19. Si OK es euclidiano, entonces también es factorial.

Demostración. Sea π irreducible. Supongamos que π | αβ y que π - α, comoπ es irreducible son únicos divisores son ±ε, ±επ donde ε es una unidad.Deducimos que ε = (α,π) = αλ′ +πµ′ (Bézout), de donde dividiendo por εse tiene 1 = αγ +πµ, luego β = αβλ+πβµ es decir π | β.

Teorema 20. K =Q(i) es euclidiano.

Demostración. Sean α,β ∈ OK , β , 0. Como β es invertible en Q(i) se tie-ne α

β = u + iv con u,v ∈ Q. Sean x e y los enteros más próximos a u y vrespectivamente. Pongamos r = u − x, s = v − y, γ = x + iy y δ = β(r + is).Tenemos que |r |, |s| ≤ 1

2 y α = βγ + δ con δ = α − βγ ∈ OK , además N (δ) =N (β)N (r + is) =N (β)(r2 + s2) <N (β) ya que r2 + s2 ≤ 1

4 + 14 < 1.

Resumiendo un poco los resultados que hemos obtenido:

1. Q(i) es euclidiano y por lo tanto factorial, en particular un elementode Q(i) es primo si y sólo si es irreducible.

2. OQ(i) = Z[i].

3. Las unidades de Z[i] son {±1,±i}.

Lema 10. Sea α ∈ OQ(i). Si N (α) ∈ {2,3,5,7,11,13, ...} (los primos), entoncesα es un primo gaussiano.

Demostración. Supongamos que α = βγ , entonces N (α) = N (β)N (γ) ⇒N (β) = ±1 o bienN (γ) = ±1

Lema 11. Sea π ∈ OQ(i) un primo gaussiano. Existe un único número primo ptal que π | p en OQ(i)

62

Demostración. Observemos que π | ππ = N (π) ∈ Z. Elijamos el más pe-queño de los enteros positivos divisibles por π, digamos p. Un tal p debeser necesariamente primo ya que si p = mn tendríamos que π | mn = p,pero π es un primo gaussiano y entonces π |m o bien π | n lo que contradi-ce la minimalidad de p, luegom = p o n = p de donde sigue que p es primo.

Para la unicidad, observemos que si existe otro primo p′ divisible por π, larelación de Bézout en Z nos dice que existen a′, a′ ∈ Z tales que ap+a′p′ = 1,luego π | ap+ a′p′ = 1 entonces π debe ser una unidad lo que no es posibleya que π es primo, sigue entonces que p = p′.

En lo que resta de esta sección K =Q(i).

Lema 12. Sea p un número primo, p es un primo gaussiano o bien el productode dos primos gaussianos conjugados p = ππ.

Demostración. Sea p número primo y π ∈ OK tal que π | p, es decir p = λπλ ∈ OK . Tenemos que p2 =N (p) =N (λ)N (π), luego si N (λ) = 1 entoncesλ es una unidad y por consiguiente p es un primo gaussiano. Si N (λ) =p, por el lema 10 λ debe ser un primo gaussiano (del mismo modo π) ynecesariamente λ = π ya que λπ ∈ Z.

Es natural preguntarse en qué caso nos encontramos dado un númeroprimo p. La respuesta está en el siguiente teorema.

Teorema 21. Los primos gaussiano son todos los números primos p ≡ 3 mod 4,los factores primos de p ≡ 1 mod 4 en p = ππ y por 1 + i.

Antes de comenzar la prueba de este teorema observemos que esta cla-sificación es "módulo una unidad", por ejemplo 1−i es también irreducible(de manera equivalente primo) pero 1− i = −i(1 + i). Ahora bien, gracias ala proposición 17 podemos ver que las únicas unidades de OK son {±1,±i}.

Demostración. Si p ≡ 3 mod 4⇒N (π) , p ya queN (π) = x2+y2 ≡ 0,1,2 mod 4.

Si p ≡ 1 mod 4 la ley de reciprocidad cuadrática nos dice que −1 es unresto cuadrático módulo p, luego p | x2 + 1 = (x − i)(x + i) para un ciertox ∈ Z. Si p fuese un primo gaussiano entonces p | x − i o bien p | x + i, enambos casos se tiene x

p ±ip < OK .

Si p = 2, 2 = (1 + i)(1 − i) = ππ. Se muestra fácilmente que 1 + i debe serprimo gaussiano.

63

7.6. Ecuaciones de Pell

Sea d ∈ Z sin factores cuadrados positivo, nos interesamos en las solu-ciones enteras de la ecuación

x2 − dy2 = 1 (5)

fácilmente vemos una de tipo (x,y) = (±1,0). Supongamos ahora que (x,y)es una solución entera positiva de la ecuación (5), es decir con x,y > 0,entonces

x − y√d =

1

x+ y√d> 0⇒ x > y

√d⇒ x − y

√d <

1

2y√d

de donde se deduce ∣∣∣∣∣√d − xy∣∣∣∣∣ < 1

2y2

Vamos a admitir dos resultados sobre las fracciones continuas (para másdetalles consultar: “A Comprehensive course in Number Theory” de Alan Ba-ker).

1. Si∣∣∣∣∣θ − pq

∣∣∣∣∣ < 12y2 entonces

p

qes un convergente de θ =

√d.

2. La fracción continua de√d es de la forma [a0, a1, · · · , am]. El número

m se llama periodo.

Notemos porpnqn

los convergentes de√d y θn los cocientes completos de

√d. Por la observación precedente existe n ∈ N tal que x = pn, y = qn,

es decir p2n − dq2

n = 1. La teoría de fracciones continuas estudiadas en elcapítulo precedente nos dice que

√d =

pnθn+1 + pn−1

qnθn+1 + qn−1(6)

de donde, utilizando pn+1qn − pnqn−1 = (−1)n, tenemos

qn√d − pn =

(−1)n

qnθn+1 + qn−1

Ahora si n es par tenemos qn√d − pn > 0, pero

p2k

q2k≤ θ =

√d entonces n

debe ser impar.

64

De hecho n debe ser un impar de la forma lm − 1 con l un entero, m elperiodo de la fracción continua de

√d. De manera equivalente l ∈ N∗ si m

est par, l ∈ 2N∗ si m es impar.

En efecto, de la ecuación (6) tenemos (pn − qn√d)θn+1 = qn−1

√d − pn−1 de

donde

(p2n − dq2

n)θn+1 = (qn−1

√d − pn−1)(pn + qn

√d) = (−1)2

√d + c, c ∈ Z

como n es impar y p2n − dq2

n = 1 sigue que θn+1 =√d + c. De otra parte

√d = a0 +

1θ1

con θ1 = θm+1 = θ2m+1, etc.

Además θn+1 = an+1 +1

θn+2⇒ an+1 = θn+1 −

1θn+2

=√d + c − 1

θn+2= a0 +

c +1θ1− 1θn+2

∈ Z. Ahora bien como 0 <1θ1,

1θn+2

< 1, necesariamente

1θ1− 1θn+2

= 0 ⇒ θ1 = θn+2 y como el periodo es m entonces n = lm − 1

(con n impar).

Veamos ahora que esta condición necesaria sobre n es también suficien-te.

Teorema 22. Todos los convergentes descritos anteriormente, es decir con nimpar de la forma ml − 1 son las soluciones de la ecuación (5).

Demostración. Para todo n = lm − 1 impar se tiene θ1 = θn+2 y entonces√d =

pn+1θn+2 + pnqn+1θn+2 + qn

=pn+1θ1 + pnqn+1θ1 + qn

. Pero√d = a0 +

1θ1⇒ θ1 =

1√d − a0

√d =

pn+1 − a0pn + pn√d

qn+1 − a0qn + qn√d

y entonces qnd + (qn+1 − a0qn)√d = (pn+1 − a0pn) +

pn√d, como

√d es irracional (sin factores cuadrados) sigue que

qnd = pn+1 − a0pn, pn = qn+1 − a0qn

finalmente a0 =pn+1 − qnd

pn=qn+1 − pn

qn⇒ p2

n − dq2n = pnqn+1 − pn+1qn = 1 (n

impar).

De manera similar se demuestra:

65

Teorema 23. La ecuación x2 − dy2 = −1 (d sin factores cuadrados) no tienesoluciones si el periodo m de

√d es par. Si m es impar, x = pn, y = qn con

pnqn

convergente de√d, n = ml − 1 (l ∈ N∗) representan todas las soluciones

positivas.

7.7. Serie de ejercicios 11

1. Mostrar que el anillo de enteros algebraicos de Q(√

3) es euclidiano.

2. Utilizar el ejercicio precedente para mostrar que existen enteros al-gebraicos α y β en Q(

√3) tales que (1 + 2

√3)α + (5 + 4

√3)β = 1.

3. Encontrar la solución positiva minimal de la ecuación x2 − 39y2 = 1.

4. Determinar si la ecuación x2 − 31y2 = −1 admite una solución utili-zando la teoría de las ecuaciones de Pell.

66

8. La conjetura ABC

Los conjuntos Z yC[t] tienen muchas cosas en común, como anillos po-demos efectuar ciertas operaciones entre sus elementos como la adición, lamultiplicación. Además ambos son euclidianos y por lo tanto factoriales,lo que nos permite escribir para todo elemento n ∈ Z:

n = εr∏i=1

pmii

con ε ∈ {±1} y los pi primos (=irreductibles). Análogamente, para todopolinomio p(t) ∈ C[t]:

p(t) = cs∏i=1

(t −αi)mi

ya que C es algebraicamente cerrado, los polinomios irreductibles son losmonomios t−α. Observemos que los αi son las raíces de p(t) con multipli-cidad mi correspondiente.

Esta analogía entre Z y C[t] es sin embargo más profunda.

8.1. ABC para los polinomios

Sea f ∈ C[t], f , 0. Escribamos f (t) = c∏ri=1(t−αi)mi con los αi distintos.

Seandeg f :=

∑i=1

mi

yn0(f ) := r.

Observemos que deg f puede ser grande aún si n0(f ) es pequeño, porejemplo si f (t) = (t −α)1000, deg f = 1000, n0(f ) = 1.

Teorema 24. Sean f ,g,h ∈ C[t] polinomios no constantes relativamente pri-mos tales que f + g = h. Entonces

max(degf ,degg,degh) ≤ n0(f gh)− 1

Antes de dar la demostración de este teorema, veamos un resultadoparticularmente interesante que se deduce de él:

67

Corolario 8. (Fermat para los polinomios) Si n ≥ 3 no hay ninguna solu-ción a la ecuación:

x(t)n + y(t)n = z(t)n

donde x,y,z ∈ C[t] polinomios no constantes relativamente primos.

Demostración. Sean f (t) = x(t)n, g(t) = y(t)n y h(t) = z(t)n. Por hipótesisf + g = h, luego por el teorema ABC para los polinomios (teorema 24)tenemos

max (degxn,degyn,degzn) ≤ n0(xnynzn)− 1 = n0(xn) +n0(yn) +n0(zn)− 1

ya que los polinomios x,y,z son relativamente primos, es decir sin raícescomunes. Ademas las relaciones degxn = ndegx, n0(xn) = n0(x) ≤ degxnos dicen que

max (ndegx,ndegy,ndegz) ≤ degx+ degy + degz − 1

de donde

n(degx+ degy + degz) ≤ 3(degx+ degy + degz)− 3

y luego(n− 3)(degx+ degy + degz)︸ ︷︷ ︸

>0

≤ −3

entonces necesariamente n < 3.

Falsos ejemplos

(1) t3 + ( 3√

2t)3 = ( 3√

3t)3. Este ejemplo no contradice el corolario 8 ya quesi dividimos por t3 para tener polinomios relativamente primeros te-nemos 13+( 3

√2)2 = ( 3

√3)2, ahora bien estos polinomios son constantes

y el corolario no se aplica.

(2) (t − a)2 + (2ta)2 = (t2 + a2)2. Este ejemplo nos muestra que como en elcaso de los números, Fermat es cierto para n ≤ 2.

(3) (x + 1)p = xp + 1p (mod p). Es decir este resultado no se aplica paratodos los cuerpos.

68

8.2. Preparativos para la prueba del teorema ABC para lospolinomios

Definición 25. Sea f una función compleja non nula, llamamos derivadalogarítmica de f a la función

L(f ) :=f ′

f

En los puntos en los que f no se anula, la derivada logarítmica coincide con lacomposición de la función |f | y la función log, es decir log◦|f | como lo muestrala regla de la derivación en cadena.

Las siguientes propiedades de la derivada logarítmica son un ejercicioelemental de verificación:

1. L(f1 · · ·fn) =n∑i=1

L(fi)

2. L(f

g

)= L(f )−L(g)

3. Si f (t) = cr∏i=1

(t −αi)mi entonces L(f )(t) =r∑i=1

mit −αi

Comencemos ahora la prueba del teorema 24

Demostración. Tenemos f + g = h ⇒f

h︸︷︷︸R

+g

h︸︷︷︸S

= 1 ⇒ R′ + S ′ = 0 ⇒

R′/RS ′/S

= −SR

, o si se prefiereL(R)L(S)

= −SR

.

Luegog

f=g/h

f /h=SR⇒g

f= −L(R)L(S)

=L(f )−L(h)L(g)−L(h)

si utilizamos la propiedad

2 de la derivada logarítmica.

Si f (t) = cf∏

(t − αi)mi , g(t) = cg∏

(t − βj)nj y h(t) = ch∏

(t − γk)lk . Segúnla propiedad 3:

g

f= −

∑ mit−αi −

∑ lkt−γk∑ nj

t−βj −∑ lk

t−γk

(7)

69

Sea D(t) =∏

(t − αi)∏

(t − βj)∏

(t − γk), tenemos que degD = n0(f gh) y

deg(D(t)t−αi

)= n0(f gh)− 1, similarmente para βj y γk.

Si multiplicamos la ecuación (7) por D(t)D(t) obtenemos:

g

f= −

(∑ mit−αi −

∑ lkt−γk

)D(t)(∑ nj

t−βj −∑ lk

t−γk

)D(t)

=Polinomio de grado ≤ n0(f gh)− 1

Otro polinomio de grado ≤ n0(f gh)− 1

Como g y f son polinomios primos relativos, necesariamente degg,degf ≤n0(f gh) − 1 y ya que h = f + g ⇒ degh ≤ n0(f gh) − 1. Finalmente hemosprobado que

max {degf ,degg,degh} ≤ n0(f gh)− 1

Existen muchas pruebas del teorema ABC para polinomios (teorema24), la mayor parte de ellas utiliza la noción de derivada.

8.3. ¿ABC para los enteros?

Primeramente tendríamos que buscar los análogos a degf y n0(f ) pa-ra los enteros. Sea m ∈ N, m =

∏ri=1p

mii . Una posible elección podría ser

’degm =∑ri=1mi ’ y n0(m) = r. Sin embargo en retrospectiva sería mejor

corresponder

degP ←→ log |m|︸︷︷︸”altura”

=r∑i=1

mi logpi︸︷︷︸”peso”

n0(P )←→ n0(m) =r∑i=1

logpi

o bien N0(m) =∏p|m

p =r∏i=1

pi = en0(m) (el radical de m).

La altura puede ser grande aún si n0 es pequeño, por ejemplo si m = 21000

tenemos log21000 = 1000log2 > n0(21000) = log2.

Conjetura ABC (Masser-Oesterlé 1986)

70

∀ε > 0, ∃C tal que ∀a,b,c enteros relativamente primos con a + b = c setiene

max {log |a|, log |b|, log |c|} ≤ (1 + ε)n0(abc) +C

o de manera equivalente

max {|a|, |b|, |c|} ≤ CN0(abc)1+ε

En agosto del 2012 el matemático japonés Shinichi Mochizuki publicó unarticulo en su página personal afirmando haber demostrado este enuncia-do. Su prueba de más de 500 páginas está pendiente pendiente de verifi-cación por otros matemáticos.

Observemos que es no es posible tener ε = 0 en general. En efecto, si to-mamos an = 32n , bn = −1 y cn = 32n − 1, tenemos que an + bn = cn con an,bny cn relativamente primos. Además si n > 0, 2n+2 | cn, luego

N0(anbncn) ≤ 3 · 2 · |cn|2n+2 ≤

3|an|2n+1

en consecuencia tenemos

max {|an|, |bn|, |cn|} = |an| > 2n+1N0(anbncn)3

> 2n−1N0(anbncn)

luego el cocientemax {|an|, |bn|, |cn|}

N0(anbncn)> 2n−1

puede tomar valores arbitrariamente grandes.

Si la conjetura ABC para los enteros es cierta podemos, entre otras cosas,deducir el último teorema de Fermat en su forma asintótica.

Corolario 9. Para n suficientemente grande, no existe ninguna solución enterano trivial de la ecuación xn + yn = zn

Demostración. Podemos suponer (caso contrario dividimos por lo que seanecesario) que si hay una solución (x,y,z), esta es tal que x,y y z son pri-mos relativos. Si por el absurdo tal solución existe, es decir xn︸︷︷︸

a

+ yn︸︷︷︸b

=

zn︸︷︷︸c

, ABC implica que

n log |z| ≤ (1 + ε) log |x||y||z|+K ≤ 3(1 + ε) log |z|+K

71

luego si n >K

log |z|+ 3(1 + ε) la desigualdad de arriba ya no es verdadera y

entonces se contradice ABC.

El interés de la conjetura ABC no sólo se concentra en la aplicaciónque acabamos de ver. La conjetura ABC implica muchos otros resultadosinteresantes en aproximación diofantina, etc. Sería interesante poder de-finir una noción de derivada sobre Z análoga a la de los polinomios parapoder traducir la prueba del teorema 24 para los enteros.

Finalicemos este curso enunciando una conjetura debida a Oesterlé de lacual el teorema ABC para los polinomio es un caso particular:

Conjetura (Oesterlé)

Sea ϕ(u,v) ∈ C[t][u,v] con degϕ ≥ 3 y todas sus raíces distintas. ExisteC tal que para todos f ,g ∈ C[t] relativamente primos

(d − 2)deg(ϕ(f ,g)) ≤ n0(ϕ(f ,g)) +C

Esta conjetura implica el teorema 24 (que ya demostramos), simplementetomamos ϕ = u2v + uv2 = u︸︷︷︸

a

v︸︷︷︸b

(u + v︸︷︷︸c

), d = 3 y C = −1 (claro está,

habría que mostrar que en tal caso C de la conjetura es tal que C ≤ −1 yrebautizarla como C = −1).

72

9. Soluciones e indicaciones de las series de ejer-cicios

9.1. Serie de ejercicios 1

1. Observemos que si p > 2 es primo, entonces p es de la forma 4k + 1 obien de la forma 4k + 3 según la división euclidiana. Supongamos por elabsurdo que los primos de la forma 4k+1 son finitos, sea {p1, ...,pn} la listade tales primos. Suponiendo que p1 = 3, sea x = 4p2...pn+3. Entonces, pi - xpara todo i = 2, ...,n, sino pi | 3. Como el producto de números de la forma4k + 1 es también de la forma 4k + 1, sigue que x debe ser divisible porun número primo p de la forma 4k + 3. Como x no es divisible por 3, p nopertenece a la lista {p1, ...,pn} y por lo tanto es un nuevo número primo dela forma 4k + 3 más grande que los precedentes.

2. Utilizar la identidad: a2k+1 + 1 = (a+ 1)(a2k − a2k−1 + a2k−2 − · · ·+ 1

).

3. (i) Escribamos m = n+ k con k ≥ 1. Tenemos que(22m − 1

)=

(22n+k−1

)=

(22n+(k−1)

− 1)(

22n+(k−1)− 1

).

Repitiendo esta factorización cuantas veces sea necesario sobre 22n+(k−1) −1deducimos que 22n + 1 | 22m −1. Sea d =

(22n + 1,22m + 1

)et máximo común

divisor de ambos números. Como d | 22m +1 y también d | 22m −1, entoncesd | 2. Luego, d = 1 ya que 22m + 1 y 22n + 1 son impares.(ii) De la demostración del teorema 2, tenemos que pn+1 ≤ p1 · · ·pn + 1.Utilizamos esa desigualdad y la relación

(x2n − 1

)= (x − 1)

n−1∏k=0

(x2k + 1

)para mostrar por inducción que pn+1 ≤ 22n + 1. Luego, observemos quex < pπ(x)+1. Sigue entonces que x ≤ 22π(x)

ya que en realidad se puede ma-yorar pn+1 con 22n , de esta manera, aplicando logaritmos se tiene el resul-tado pedido.

4. El mínimos común múltiplo de dos números a y b es el número máspequeño que es divisible por a y b. Si a = pα1

1 · · ·pαkk , b = pβ1

1 · · ·pβkk , entonces

mcm(a,b) = pmax(α1,β1)1 · · ·pmax(αk ,βk)

k .

73

Utilizar luego la observación 2.

5. Mostrar por inducción que si n ≥ 2, entonces Hn = anbn

con an impar ybn par no nulos.

9.2. Serie de ejercicios 2

1. Sean {p1, ...,pk} los factores primos distintos de n. La función µ solo con-tabiliza la contribución de aquellos divisores de n donde no se repita unpi . Así, el divisor pi1 · · ·pil , contribuye con |µ(pi1 · · ·pil )| = |(−1)l | = 1 y hayen total

(kl

)de ellos en n. Luego∑

d|n|µ(d)| = 1 +

(k1

)+(k2

)+ · · ·+

(kk

)= 2k .

2. Las funciones µ y σ son multiplicativas, luego µσ es multiplicativa y porlo tanto g(n) :=

∑d|nµ(d)σ (d) es multiplicativa por el lema 3. Entonces, sólo

necesitamos conocer el valor de g en una potencia de un primo pj :

g(pj

)= µ(1)σ (1) +µ(p)σ (p) = 1− (1 + p) = −p.

Finalmente, si n = pα11 · · ·p

αkk , se tiene

g(n) = (−1)kp1 · · ·pk .

3. Por el lema 5, tenemos Id(n) =∑d|nϕ(n), luego por la formula de inver-

sión de Möbius ϕ(n) =∑d|nµ(d)Id

(nd

)⇒ϕ(n)n

=∑d|n

µ(d)d

.

Si Re s > 1, tenemos ∞∑n=1

µ(n)ns

∞∑n=1

1(n)ns

︸ ︷︷ ︸ζ(s)

=∞∑n=1

(µ ∗1)(n)ns

lema 4=∞∑n=1

ε(n)n

= 1,

lo que nos da la relación∞∑n=1

µ(n)ns

=1ζ(s)

. (8)

74

Necesitaremos también la siguiente estimación∑d≤x

1d

= logx+ O(1). (9)

Para una justificación de la fórmula 9, comparar la suma de la izquierdacon la integral

∫ x1duu .

Ahora ya disponemos de todos los ingredientes para este ejercicio:∑n≤x

ϕ(n)n

=∑n≤x

∑d|n

µ(d)d

=∑d≤x

∑m≤ xd

µ(d)d

=∑d≤x

µ(d)d

∑m≤ xd

1

︸︷︷︸[x/d]

=∑d≤x

µ(d)d

(xd− {xd})

= x∑d≤x

µ(d)d2 + O(1)

∑d≤x

µ(d)d.

Para la primera suma tenemos∑d≤x

µ(d)d2 =

∞∑d=1

µ(d)d2 + O

(1x

)(8)=

1ζ(2)

+ O(1x

)=

6π2 + O

(1x

).

Y para la segunda∣∣∣∣∣∣∣∑d≤x

µ(d)d

∣∣∣∣∣∣∣ ≤∑d≤x

1d

(9)= logx+ O(1)⇒

∑d≤x

µ(d)d

= O(logx).

Finalmente ∑n≤x

ϕ(n)n

=6π2x+ O(logx).

4. i) Sea n = pα11 · · ·p

αkk la descomposición de n es factores primos distintos.

Solamente los divisores de n de la forma pα contribuyen a la suma conlogp. Por cada pi que divide n, los divisores pi ,p

2i , ...,p

αii contribuyen con

αi logp, luego∑d|n

Λ(d) =k∑i=1

αi logpi = log(pα1

1 · · ·pαkk

)= logn.

En particular, hemos mostrado que Λ ∗1 = log.

ii) Observemos que∑n≥1

Λ(n)ns =

∑p∑m≥1

logppms . Por el teorema 4, tenemos

ζ(s) =∏p

(1− p−s)−1

75

si Re s > 1. Luego logζ(s) =∑p log(1− p−s)−1 = −

∑p∑m≥1

p−ms

m . Derivandoambos miembros, tenemos

ζ′(s)ζ(s)

=∑p

∑m≥1

logppms

=∑n≥1

Λ(n)ns

.

9.3. Serie de ejercicios 3

1. Podemos suponer |x| < 1 y observar que la derivación término a términode la serie

∑∞n=1x

n nos da ∑n≥1

nxn =x

(1− x)2 . (10)

La fórmula (10) puede ser considerada como una igualdad formal de po-linomios, luego no es necesario poner una condición sobre x. Tenemos en-tonces

∞∑n=1

ϕ(n)xn

1− x2 =∞∑n=1

ϕ(n)

∞∑k=1

(xn)k =

∑m≥1

∑d|m

ϕ(d)

︸ ︷︷ ︸m

xm

=∑m≥1

mxm =x

(1− x)2 .

2. Multiplicando cada ecuación por las inversas de 2,3 y 5 módulo 3,5 y 7respectivamente, transformamos el sistema de congruencias en el sistemaequivalente

x ≡ 2 (mod3)x ≡ 2 (mod5)x ≡ 3 (mod7)

Para poder aplicar el teorema chino del resto (teorema 5) debemos calcularlos valores: M1 = 5 · 7⇒ y1 = 2, M2 = 3 · 7⇒ y1 = 1 y M3 = 3 · 5⇒ y1 = 1.Luego la solución es

x = y1M1c1 + y2M2c2 + y3M3c3 = 140 + 42 + 45 = 122 ≡ 17 (mod105).

3. Ya que k ≥ 1, akp−k+1 ≡ a (modp) es una consecuencia directa del peque-ño teorema de Fermat (teorema 7).

76

Sea 798 = 2 · 3 · 7 · 19 la descomposición en factores primos de 798. Obser-vemos que para cada divisor primo p de 798, existe un entero k ≥ 1 tal quekp − k + 1 = 19. Por ejemplo, para p = 7 tenemos k = 3. Luego, por lo vistoanteriormente p | akp−k+1−a = a19−a. Deducimos entonces que 798 | a19−a.

4. Buscamos enteros a,b,c,d y e tales que para todo entero x, al menosuna de las congruencias:

x ≡ a (2) (A)x ≡ b (3) (B)x ≡ c (4) (C)x ≡ d (6) (D)x ≡ e (12) (E)

sea satisfecha. Como nuestra elección es arbitraria, podemos empezar po-niendo a = 0 (esto supone quitar todos los x impares). Luego podemosponer b = 0, enseguida podemos poner c = 1 (o bien c = 3, pero no 2 ni4 porque x satisfacería la ecuación (A) y lo que queremos es cubrir todaslos posibles valores de x para que se verifique al menos una de las con-gruencias). Si x = 0,2,4 (6), x satisface (A) y si x = 3 (6), x satisface (B). Enconsecuencia, es mejor poner d = 1 o d = 5.

Finalmente, para hacer la mejor elección de e, elegimos un valor que nosatisfaga alguna de las otras congruencias. Así, si e = 0,2,4,6,8,10 (12) en-tonces x verifica (A), si e = 3,9 (12) entonces x verifica (B), si e = 1,5 (12)entonces x verifica (C) y si e = 7 (12) entonces x verifica (D). Luego, lo me-jor es poner e = 11. Vemos entonces que hemos logrado cubrir todos loscasos.

9.4. Serie de ejercicios 4

1. Ver el ejercicio 2 de la serie 3.

2. Podemos suponer sin perdida de generalidad que a1 = 0 y b1 = 0. Su-pongamos por el absurdo que {a2b2, ..., apbp} ∪ {0} es un sistema completode residuos módulo p, por el teorema de Wilson (proposición 8) tenemosque

p∏i=2

aibi ≡ −1 (modp).

77

De la misma forma,∏pi=2 ai =

∏pi=2 bi ≡ −1(modp). Luego

∏pi=2 aibi ≡ 1(modp),

lo cual no es posible.

3. Observemos que 1,2, ...p−12 ,−1,−2, ...,−p−1

2 son todos los residuos módulop no nulos, luego por la proposición 8:

(−1)p−1

2 x2 = (p − 1)! ≡ −1 (modp),

de dondex2 ≡ (−1)

p+12 (modp).

4. Observemos que la indicatriz de Euler posee la propiedad siguiente

Sea n > 0 un entero,ϕ(n) = n− 1⇔ n es primo. (11)

Por hipótesis a , 0. Como n > 2 y a ·an−2 ≡ 1(modn), sigue que a ∈ (Z/nZ)∗.Luego, por la proposición 6, aϕ(n) ≡ 1 (modn). Sea r el orden de a, pordefinición de orden r | n−1. Por hipótesis r = n−1 y r = n−1 | ϕ(n). Ya queϕ(n) ≤ n− 1, necesariamente ϕ(n) = n− 1 y luego, por (11), n es primo.

9.5. Serie de ejercicios 5

1. Observemos que 3 es un elemento primitivo en (Z/7Z)∗ ya que 31 = 3,32 = 2, 33 = 6, 34 = 4, 35 = 5 y 36 = 1. Tomando logaritmos discretos enesta base:

2logy ≡ log5 + 3logx (mod6)2logy ≡ 5 + 3logx (mod6).

Poniendo X = logx y Y = logy obtenemos

(X,Y ) ∈ {(1,1), (1,4), (3,1), (3,4), (5,1), (5,4)}.

De donde derivamos las soluciones de la ecuación original:

(x,y) ∈ {(3,3), (3,4), (6,3), (6,4), (5,3), (5,4)}.

2. Sea g un elemento primitivo de (Z/pZ)∗. Tenemos que

p∑n=1

nk ≡p−1∑l=1

(gk)l (modp).

78

Luego si p − 1 | k, entonces gk ≡ 1 (modp) según la proposición 7. Por lotanto

p−1∑l=1

(gk)l ≡

p − 1 ≡ −1 (modp) si p − 1 | k(gk)p−gkgk−1

≡ 0 (modp) sino.

3. Con las mismas notaciones de la sección 4.3, la lista de los represen-tantes más pequeños de j · 2 es

{2,4,6, ...,2

[p−1

4

],2

([p−1

4

]+ 1

)− p, ...,−3,−1

},

luego l = p−12 −

[p−1

4

]. Entonces, l es par si p ≡ ±1 (mod8) y en ese caso(

2p

)= 1, l es impar si p ≡ ±3(mod8) y en tal caso

(2p

)= −1. Lo que podemos

resumir escribiendo: (2p

)= (−1)

p2−18 .

4. Aplicando las propiedades del símbolo de Legendre y de la reciprocidadcuadrática:(15

73

)=

( 373

)( 573

)=

(735

)(−1)

73−12 ·

5−12

(733

)(−1)

3−12 ·

73−12

=(35

)(13

)=

(53

)=

(23

)= (−1)

32−18 = −1.

5. Si p = 4k + 3, entonces p′ = 2p + 1 = 8k + 7. Luego por el corolario 3,(2p′

)= 1. Existe entonces a no divisible por p′ tal que 2 ≡ a2 (modp′). Luego

por el pequeño teorema de Fermat:

2p ≡ a2p ≡ ap′−1 ≡ 1 (modp′).

Como p = 251 = 6 · 4 + 3 es primo y p′ = 2p + 1 = 503 también lo es. Luegop′ = 503 | 2251 − 1. Por lo tanto 2251 − 1 no es primo.

9.6. Serie de ejercicios 6

1. Supongamos que p , 2,5 (en ambos casos 5 es un residuo cuadrático).Por la ley de reciprocidad cuadrática(

5p

)(p5

)= (−1)

5−12 ·

p−12 = 1.

Luego 5 es un residuo cuadrático módulo p si y solamente si p es un resi-duo cuadrático módulo 5. Por el criterio de Euler (proposición 11):(p

5

)≡ p

5−12 = p2 (mod5).

79

Tenemos que determinar entonces los primos p tales que p2 ≡ 1 (mod5).Calculando módulo 10 deducimos que(

5p

)=

{1 si p ≡ 1,9 (mod10)−1 si p ≡ 3,7 (mod10)

2. Aplicando las propriedades se obtiene(

123917

)= 1.

3. Si p = 2n+1 es primo, entonces g es primitivo si y solamente si(gp

)= −1.

⇒) Supongamos por el absurdo que q ≡ x2 (modp). Como g es primitivo,existe un entero l tal que x ≡ g l (modp). Luego se tendría g2l−1 ≡ 1 (modp)y entonces ord(g) = p − 1 = 2n | 2l − 1 lo cual no es posible.⇐) La contrapuesta es una reformulación de la proposición 11.

Supongamos que n > 1. Entonces p = 2n + 1 > 3 es un primo impar. Porotra parte, la ley de reciprocidad cuadrática nos dice que(

3p

)(p3

)= (−1)

(2n+1)−12 (−1)

3−12 = 1,

es decir 3 es un residuo cuadrático módulo p si y solamente si p = 2n+ 1 esun residuo cuadrático módulo 3. Para deducir que 3 es un elemento primi-tivo módulo p, será suficiente mostrar que p no puede ser un residuo cua-drático módulo p. Supongamos por el absurdo que p = 2n + 1 ≡ x2 (mod3).Observemos que 3 - x y que todos los cuadrados no divisibles por 3 soncongruentes a 1 módulo 3. Por otra parte, para todo n ≥ 0, 2n ≡ 1,2(mod3),lo que implica que 2n + 1 ≡ 2,0 (mod3). Deducimos entonces que p no esun cuadrado módulo 3 y por lo visto anteriormente 3 es un elemento pri-mitivo módulo p.

4. Como a . 0 (modp) y p > 2, a y 2 son invertibles módulo p. Luego la

ecuación ax2 + bx + c ≡ 0 (modp) es equivalente a(x+ b

2a

)2≡ d

4a2 (modp).Por el teorema de lagrange esta ecuación tiene a lo sumo 2 soluciones yde hecho posee exactamente dos soluciones si y sólo si d es un cuadradomódulo p.

5. Sea la afirmación A : “Si n ≥ 6, entonces n = p+q+r con p, q y r primos”.Denotemos por G la conjetura de Goldbach G : “Todo numero par mayor oigual a 4 se escribe como suma de dos primos”. Entonces A⇔ G. En efecto,G⇒ A) Si n ≥ 6 es impar, entonces n−3 ≥ 4 es par y por G, n = 3+p+q con

80

p y q primos. Si n ≥ 6 es par, entonces n−2 ≥ 4 es par y por G, n = 2 +p+qcon p y q primos.A⇒ G) Si n ≥ 4 es par, entonces por A, n+ 2 = p+ q+ r con p,q y r primos.Como n+2 es par, p,q y r no pueden ser simultáneamente pares, luego seaq = 2, sea p = 2, sea r = 2. Finalmente, podemos suponer sin pérdida degeneralidad que n = p+ q, lo que muestra G.

9.7. Serie de ejercicios 7

1. g(x,y) = x2 + 4xy + 5y2 = (x + 2y)2 + y2 = f (x + 2y,y), es decir g = f ◦ γ

con γ =(1 20 1

)∈ SL2(Z). Luego los enteros de la forma x2 + 4xy + 5y2 son

todos los enteros positivos que se escriben como suma de dos cuadrados,los cuales están caracterizados en el teorema 9.

2. Sea n = 5 y definamos las formas cuadráticas siguientes

f (x1, ...,x5) =5∑i=1

x2i , g(x1, ...,x5) =

4∑i=1

x2i + 2x2

5.

detMf = 1, detMg = 2⇒ f � g. Las imágenes de f y g son enteros posi-tivos y el teorema 10 de la suma de cuatro cuadrados (poniendo x5 = 0)nos dice que son exactamente todos los enteros positivos, es decir f (Z2) =g(Z2) = Z≥0.

3. Los cuadrados son congruentes a 0,1,4 módulo 8, luego la suma de trescuadrados es congruente a 0,1,2,3,4,5,6 módulo 8. Deducimos entoncesque {8l + 7 | l ∈ Z≥0} ⊂ E. Por lo tanto

lımsupx→∞

1x|{a ≤ x | a ∈ E}| ≥ lımsup

x→∞

1x|{0 ≤ α ≤ x | α ≡ 7(8)}|

≥ lımsupx→∞

1x

[x+ 78

]≥ lımsup

x→∞

1x

(x+ 78−{x+ 7

8

})=

18

4. Supongamos por el absurdo que 22k+1 = a2 + b2 + c2 + d2 con a ≥ b ≥ c ≥d > 0 enteros. Como 22k+1 ≡ 0 (4), a,b,c,d ≡ 0 (2) o bien a,b,c,d ≡ 1 (2).El segundo caso, es decir cuando a,b,c y d son impares no es posible ya

81

que a2 + b2 + c2 + d2 ≡ 4 (8), lo que contradice el hecho que 22k+1 ≡ 0 (8)(k ≥ 1). Finalmente el primer caso se reduce al segundo dividiendo por 4la igualdad 22k+1 = a2 + b2 + c2 + d2 tantas veces como sea necesario paraque a,b,c y d sean todos impares, es decir hasta que k sea igual a 1.

9.8. Serie de ejercicios 8

1. Sea θ = 3,1415926. Aplicamos el algoritmo comenzando con θ0 = θ ya0 = [θ0] = 3:

θ1 =1

θ0 − a0= 7,06251597894240⇒ a1 = [θ1] = 7,

θ2 =1

θ1 − a1= 15,9959104362960⇒ a2 = [θ2] = 15,

θ3 =1

θ2 − a2= 1,00410635691218⇒ a2 = [θ3] = 1.

Luego los primeros cuatro convergentes de θ son

p0

q0= 3,

p1

q1=

227,p2

q2=

333106

,p3

q3=

355113

.

2. Sea θ = [a0, ..., an] con a0 ≥ 0. Tenemos que 1θ < 1 ya que ai ≥ 1 para todo

i = 0, ...,n. Luego b0 =[

]= 0 y seguidamente θ1 = 1

1/θ−b0= θ, de donde

b1 = [θ1] = a0 y por lo tanto 1θ = [b0,b1, ...,bn+1] = [0, a0, a1, ..., an].

3. Como n <√n2 + 1 < n+ 1, entonces a0 = [

√n2 + 1] = n. Luego, aplicando

el algoritmo:

θ1 =1

θ − a0=

1√n2 + 1−n

=√n2 + 1 +n⇒ a1 = [θ1] = 2n,

θ2 =1

θ1 − a1=

1

(√n2 + 1 +n)− 2n

=√n2 + 1 +n⇒ a2 = [θ2] = 2n,

vemos que la sucesión ai se vuelve constante igual a 2n a partir de i ≥ 1.Por lo tanto

√n2 + 1 = [n,2n].

4. Sea p(x) = x2 − ax − 1 con a ∈ N∗. La ecuación p(x) = 0 posee dos raí-ces reales diferentes, una estrictamente positiva θ1 > 0 y la otra θ2 < 0estrictamente negativa. Para la raíz positiva se tiene

θ21 − aθ1 − 1 = 0⇔ θ1 = a+

1θ1

= a+1

a+1

a+ · · ·

.

82

Luego θ1 = [a,a, ...] = [a]. Refiriéndonos a la demostración del teorema 13,tenemos la relación qn = aqn−1 + qn−2 con q0 = 1 y q1 = a (observar quep1q1

= a2+1a , de donde q1 = a ya que (a2 + 1)− a(a) = 1 y entonces por la pro-

posición 2, a2 + 1 y a son primos relativos). Hay esencialmente 2 manerasde resolver la ecuación de recurrencia qn = aqn−1 + qn−2.

Primera forma: Considerar la relación matricial(qnqn−1

)=

(a 11 0

)(qn−1qn−2

)que iterándola nos da (

qnqn−1

)=

(a 11 0

)n−1 (q1q0

),

luego diagonalizamos la matriz en cuestión para evaluar su (n − 1)-ésimapotencia y así poder calcular qn.

Segunda forma: Buscar una solución de la forma qn = Arn con A una cons-tante que determinaremos con las condiciones iniciales q0 = 1,q1 = a. Rem-plazando qn por Arn en la ecuación de recurrencia tenemos r2 − r − 1 = 0.Esta ecuación tiene como raíces θ1 y θ2. La homogeneidad de la ecuaciónde recurrencia implica que qn = Aθn1 +Bθn2 es la solución general. Con lascondiciones iniciales determinamos A = − θ1

θ2−θ1, B = θ2

θ2−θ1. Finalmente

qn =θn+1

1 −θn+12

θ1 −θ2.

La sucesión de Fibonacci está definida por F0 = 0,F1 = 1,Fn = Fn−1 + Fn−2para n ≥ 2. Luego poniendo qn−1 = Fn, a = 1 obtenemos

Fn =1√

5(ϕn − ϕn),

donde ϕ es la raíz positiva de x2 − x−1 = 0 (conocido popularmente comoel número de oro) y ϕ = − 1

ϕ .

9.9. Serie de ejercicios 9

1. Sea θ = [1,2,3,4]. Pongamos θ = [3,4]. Tenemos que

θ = 1 +1

2 + θ=

3 + θ2 + θ

.

83

Luego θ = 3 + 14+1/θ

⇔ θ2 = 12, de donde θ =√

12.

2. Sea θ = [a0, a1, ...] irracional y ϕ = 1+√

52 . Tenemos que mostrar que qn ≥

ϕn−1. Procedemos por inducción sobre n:

i. n = 0,n = 1 se verifican trivialmente.

ii. n⇒ n+ 1 : ϕ satisface ϕ2 −ϕ − 1 = 0, luego

qn+1 = an+1qn + qn−1 ≥ ϕn−1 +ϕn−2 = ϕn−2 (ϕ + 1)︸ ︷︷ ︸ϕ2

= ϕn.

Supongamos ahora que an ≤ A para todo n. El número θ = 12(A+

√A2 +A)

satisface θ2−Aθ−1 = 0. Luego un razonamiento por inducción como antespermite de concluir que qn ≤ θn.

3. Sea θ = [a0, a1, ...] con ai ≥ ai+1 para todo i = 0,1,2, .... Mostremos pri-mero por inducción que qn ≤ (an + 1)n para todo n. Los casos n = 0,n = 1se verifican trivialmente. Supongamos que qk ≤ (ak + 1)k para todo k ≤ n,entonces

qn+1 = an+1qn + qn−1 ≤ an+1 ≤ an+1( an︸︷︷︸≤an+1

+1)n + ( an−1︸︷︷︸≤an+1

+1)n−1

≤ (an+1 + 1)n− (an+1(an+1 + 1) + 1)︸ ︷︷ ︸≤(an+1+1)2

≤ (an+1 + 1)n+1.

Si además an+1 ≥ (an + 1)n2

para todo n, entonces θ es trascendente. Enefecto, supongamos por el contrario que θ sea algebraico de grado d. Porel teorema de Liouville, existe una constante c = c(θ) > 0 tal que∣∣∣∣∣θ − pnqn

∣∣∣∣∣ ≥ c

qdn.

Por otra parte, qn+1 = an+1qn + qn−1 > an+1qn y luego por el teorema 13, setiene también ∣∣∣∣∣θ − pnqn

∣∣∣∣∣ ≤ 1qnqn+1

<1

an+1q2n≤ 1

q2+nn

.

84

La última desigualdad es una consecuencia de la hipótesis adicional: an+1 ≥(an + 1)n

2 ≥ qnn. En resumen tenemos

c

qdn≤

∣∣∣∣∣θ − pnqn∣∣∣∣∣ ≤ 1

q2+nn

,

de donde c ≤ qd−(2+n)n para todo n, lo cual es posible solamente si c = 0.

Esto contradice el teorema de Liouville, por lo tanto θ es trascendente.

4. Sea θ = a−b + a−b2

+ a−b3

+ · · · . El criterio del cociente nos dice que θ ∈ R.Sean

θm = a−b + a−b2

+ · · ·+ a−bm−1∈Q,

qm = abm−1∈ N, pm = qmθm ∈ N.

Sea ε = δb con δ > 0 tal que u := 2b + δ < 1. Esto es posible ya que b ≥ 3.

Luego, para todo c > 0 podemos encontrar m tal que

a−bm

+ a−bm+1

+ · · · < ca−ubm.

En efecto, para todo m ≥ 1, a−bm

+ a−bm+1

+ · · · converge hacia un númeroreal acotado por θ. Como u < 1, para todo c > 0 se puede elegir m ≥ 1 talque θ < ca−ub

m. Finalmente, existe una infinidad de m tales que

|θ −θm| =∣∣∣∣∣θ − pmqm

∣∣∣∣∣ ≤ c

a(2+δb)bm−1 ≤c

q2+δbm

≤ c

qε+2m

.

Luego por el corolario 4, θ es irracional. Además θ no puede ser algebrai-co, sino se contradicería el teorema de Roth. Por lo tanto θ es trascendente.

9.10. Serie de ejercicios 10

1. Como 2 ≡ 2 (mod4), se tiene OQ(√

2) = Z[√

2]. Para que x + y√

2 ∈ Z[√

2]

sea una unidad, es suficiente y necesario que N (x + y√

2) = x2 − 2y2 = ±1tenga solución en los enteros. Luego de unos pocos ensayos, obtenemosuna solución (x,y) = (3,2). Finalmente, los enteros algebraicos (3 + 2

√2)n,

n ≥ 0 nos proporcionan una infinidad de unidades diferentes. De maneraanáloga, para OQ(

√3) = Z[

√3] los enteros algebraicos (2 +

√3)n, n ≥ 0 nos

dan una solución.

2. SeaK =Q(√−6). Como −6 ≡ 2(mod4),OK = Z[

√−6]. Mostremos que 2 es

85

irreducible en este anillo: Si 2 = αβ⇒ 4 =N (α)N (β) de dondeN (α) = ±2o N (α) = ±4. En el segundo caso N (β) = ∓1 y entonces β es una unidad.El el primer caso, la ecuación x2 + 6y2 = ±2 no tiene solución en los ente-ros. Sigue entonces que 2 es irreducible. De forma similar se prueba que3 y√−6 son irreducibles. Queda por ver que no están asociados entre si.

Por ejemplo 2 y√−6 no están asociados, sino existiría una unidad ε tal

que 2 = ε√−6, luego pasando a normas 4 = N (ε)6 y entonces ε no sería

una unidad. Finalmente, hemos probado que la descomposición en facto-res irreducibles no es única (módulo una unidad) y por lo tanto Z[

√−6] no

es factorial.

3. Por ejemplo, 2 no es irreducible en OQ(√

3). De hecho todos los facto-

res de izquierda y derecha no son irreducibles. En efecto, sean α = −1+√

3y β = 4 + 3

√3, entonces

αα = −2, ββ = −11, αβ = 5 +√

3, αβ = 5−√

3.

Luego 2 · 11 = (−αα)(−ββ) = (αβ)(αβ) = (5 +√

3)(5−√

3).

4. Sea K = Q(√

10), entonces OK = Z[√

10]. Observemos que para todoentero a se tiene que a2 ≡ 0,1,4 (mod5) y por lo tanto no existen ente-ros algebraicos en OK con norma ±2,±3 ya que una solución entera dex2 − 10y2 = ±2,±3 implicaría una solución módulo 5. Por otra parte el en-tero algebraico 4+

√10 es irreducible ya que no hay enteros algebraicos en

OK con norma ±2. Este mismo razonamiento muestra que 4−√

10 tambiénes irreducible. Consideremos ahora el producto

(4 +√

10)(4−√

10) = 6 = 2 · 3.

Por los motivos anteriormente mencionados 2 y 3 son irreducibles, luegoOK no es factorial.

9.11. Serie de ejercicios 11

1. Con las mismas notaciones del teorema 20 pero esta vez con K =Q(√

3)tenemos α = βγ +δ, δ = β(r+

√3s) ∈ OK = Z[

√3]. Luego es suficiente obser-

var que∣∣∣r2 − 3s2

∣∣∣ ≤max{r2,3s2} ≤ 34 < 1.

2. Sean A = 5 + 4√

3, B = 1 + 2√

3. Entonces A/B = 19/11 + 6/11√

3, luegoelegimos r = 2 y s = 1 que son los enteros más próximos a 19/11 y 6/11 res-pectivamente. Sea Γ = 2+

√2. Entonces BΓ = 8+5

√3 y ∆ := A−BΓ = −3−

√3.

86

Por construcción N (∆) < N (B) y ∆ es el resto de la división euclidianade A y B. Continuando la división tenemos B/∆ = 1/2 − 5/6

√3. Poniendo

E = −√

3 y Z = B−∆E = −2−√

3, tenemos por construcción B = ∆E +Z conN (Z) <N (∆). De hecho N (Z) = 1, lo que significa que Z es una unidad yentonces detenemos las divisiones sucesivas. Recapitulando

Z = B−∆E= B− (A−BΓ )E= (1 + ΓE)B−EA.

Por lo tanto

1 = Z−1(1 + ΓE)B−Z−1EA

1 = (−2 + 2√

3)B+ (3− 2√

3)A.

3. Primeramente calculamos√

39 = [6,4,12], de donde el periodo m = 2.Sabemos por el teorema 22 que todas las soluciones x2 − 19y2 = 1 estándadas por los convergentes de (pn,qn) tales que n es impar de la forman = ml − 1 = 2l − 1. Luego la solución positiva minimal esta dada por(x,y) = (p1,q1) = (25,4).

4. Calculamos√

31 = [5,1,1,3,5,3,1,1,10]. Vemos entonces que el perio-do m = 8 es par, por lo tanto la ecuación x2−31y2 = −1 no tiene solucionessegún el teorema 23.

87